PANCE PREP QUESTIONS Flashcards
A 15-year-old male with a recent history of camping presents after 5 days of flatulence and greasy, foul-smelling diarrhea. The patient also reports nausea, weight loss, and abdominal cramps which precedes the sudden diarrhea. He denies tenesmus, urgency, or bloody diarrhea. What is the patient most likely to report about his camping activities?
A. Collecting water from a stream, without boiling or chemical treatment
B. Recent antibiotic prescription
C. Symptoms have been going on for months
D. The patient camped as a side-excursion from a cruise ship
E. The patient camped in Mexico
Answer: A. Collecting water from a stream, without boiling or chemical treatment
EXPLANATION
Giardiasis is an infection of Giardia lamblia, a protozoan parasite that can cause epidemic or sporadic diarrhea. The major site of infection is the small intestine. While the exact etiology is uncertain, diarrhea may be a result of both intestinal malabsorption and hypersecretion. The small intestine is the site of the major structural and functional abnormalities associated with giardiasis
Giardia lamblia is a significant cause of diarrhea in hikers and campers in wilderness areas, who drink water that has not been adequately boiled or filtered.
The most common symptoms of acute giardiasis are the sudden onset of diarrhea, malaise, foul-smelling fatty stools, abdominal cramps, flatulence, nausea, and weight loss. Vomiting and fever are less common symptoms. Symptoms usually present after a 7-14 day incubation period and may last from 2-4 weeks. Complete recovery can take many weeks, even after clearance of the parasite. [3]
A 67 year-old male who was recently diagnosed with colorectal cancer presents with fever and shortness of breath that started 2 days prior to presentation. Patient also noticed that his hands are “more red than usual” and there are painful lesions on his fingers. Cardiovascular examination is remarkable for a holosytolic, “blowing” murmur best heard in the mitral area which was not present during prior examinations. Preliminary results of the blood culture is positive for a gram positive cocci. Which of the following organism is most likely responsible for patient’s disease?
A. Staphylococcus aureus B. Streptococcus viridians C. Staphyloccus epidermidis D. Streptococcus bovis E. Eikenella corrodens
Answer: D. Streptococcus bovis.
The hand erythema is likely Janeway lesions (erythematous non-tender lesions on palms and soles) and the tender lesions on the fingers are Osler’s nodes. Given the above findings and a new murmur on physical examination, the most likely diagnosis for this patient is infective endocarditis. Endocarditis is inflammation of the endocardium that lines the surface of cardiac valves; usually due to bacterial infection. Streptococci and staphylococci account for majority (80%) of cases of infective endocarditis. Common organisms responsible for infective endocarditis include the following:
Streptococcus viridians (Choice B) is the most common overall cause. It is a low-virulence organism that infects previously damaged valves (e.g. chronic rheumatic heart disease and prolapsed mitral valve).
Staphylococcus aureus (choice A) is the most common cause in IV drug abusers. S. aureus is a high-virulence organism that infects normal valves, most commonly the tricuspid valve.
Staphyloccus epidermidis (choice C) is associated with endocarditis of prosthetic valves.
Streptococcus bovis (Choice D) is associated with endocarditis in patients with underlying colorectal carcinoma. Given our patient was recently diagnosed with colorectal cancer, the most likely causative organism is S. bovis.
A 46-year-old woman wanders into the clinic, rambling incoherently. When questioned, she has
some difficulty remembering what she was asked. She exhibits some perceptual disturbances
and is not oriented to time. Which of the following is the most likely diagnosis?
(A) Bipolar disorder (B) Delirium (C) Major neurocognitive disorder (dementia) (D) Paranoid personality disorder (E) Schizophrenia
Answer: B - Delirium
Six days ago, a 2-year-old boy had a temperature of 40.0°C (104.0°F). No specific cause was
found. His fever has persisted, and he now has injected conjunctivae, strawberry tongue, dry
fissured lips, erythema and desquamation of his hands and feet, and bilateral cervical
adenopathy. Which of the following is the most likely complication of this condition?
(A) Chorea (B) Heart failure (C) Coronary artery aneurysm (D) Mesenteric arteritis (E) Valvular heart disease
Answer: C - Coronary Artery Aneurysm = deduce that the patient has Kawasaki syndrome: most common complication = coronary artery aneurysm
- Kawasakis = think warm CREAM
⦁ fever (warm)
⦁ Conjunctivitis (bilateral & nonexudative) +/- photophobia
⦁ Rash = polymorphous
⦁ Extremity changes (desquamation - especially perineum), edema, erythema of palms/soles, arthritis
⦁ Adenopathy = cervical lymphadenopathy
⦁ Mucous membrane = lip swelling, lip fissures, strawberry tongue - Unidentified respiratory agent or viral pathogen that has a propensity towards vascular tissue –> necrotizing vasculitis –> coronary vessel arteritis, CORONARY ARTERY ANEURYSM, MI, pericarditis, myocarditis, peripheral arterial occlusion
- TX = IVIG + high dose aspirin; steroids if refractory
A 31-year-old African American woman has had worsening malaise, dyspnea, and low-grade
fever for the past four weeks. She was recently treated for iritis. On physical examination, red
nodules over the anterior lower legs and mild hepatomegaly are noted. Chest x-ray study shows
bilateral hilar adenopathy. Which of the following is the most likely diagnosis?
(A) Lymphoma (B) Mesothelioma (C) Sarcoidosis (D) Tuberculosis (E) Wegener granulomatosis
Answer: Sarcoidosis
SARCOIDOSIS
- multisystemic inflammatory disorder of unknown etiology;
- INCREASED PREVALENCE IN AFRICAN AMERICANS, NORTHERN EUROPEANS, FEMALES
- 50% = asymptomatic
- NONCASEATING GRANULOMA
- most common complications = pulmonary
⦁ pulmonary: dry, nonproductive cough, dyspnea, chest pain
⦁ lymphadenopathy - hilar nodes - intrathoracic lymphadenopathy
⦁ skin = 2nd most common organ involved with sarcoidosis = ERYTHEMA NODOSUM = bilateral tender red nodules on anterior legs; may also be seen with coccidiomycosis; maculopapular rash, LUPUS PERNIO = raised discoloration of nose/ear/cheek/chin - resembles frost bite
⦁ PAROTID GLAND ENLARGEMENT
⦁ VISUAL = ANTERIOR UVEITIS, blurred vision, conjunctivitis
⦁ Myocardial = arrhythmias, cardiomyopathies
⦁ Rheumatologic = arthralgias, fever, malaise, weight loss, hepatosplenomegaly
⦁ Neurologic = cranial nerve palsies (usually CN7 - facial nerve)
⦁ HYPERCALCEMIA, increased ACE levels, Calcinosis Cutis
TX = observation; majority of patients have spontaneous remission within 2 years and require no treatment
- ORAL STEROIDS = treatment of choice when treatment is needed
For the past three weeks, a 47-year-old man has had the feeling of heaviness in his chest while
pushing his lawn mower. He says he has never been treated for similar symptoms and has no
symptoms now. He smoked one pack of cigarettes daily for 20 years but quit smoking seven
years ago. Findings on physical examination are normal. Electrocardiography shows no
abnormalities. Which of the following is the most appropriate initial diagnostic study?
(A) Coronary arteriography (B) Echocardiography (C) Exercise stress test (D) 24-Hour ambulatory cardiac monitoring (E) Myocardial perfusion scan
Answer: C - exercise stress test
A 32-year-old woman has had a rash on her legs for the past three weeks. There are no known infectious contacts. One month ago, she had a bladder infection and was treated with
trimethoprim-sulfamethoxazole. Physical examination shows a diffuse rash on the shins, the left medial ankle, and the right medial calf. The rash is tender, diffuse, and recurs in the same areas. A few of the lesions have the appearance of bruising. Which of the following is the most likely diagnosis?
(A) Erythema multiforme (B) Erythema nodosum (C) Lichen planus (D) Lichen simplex (E) Nummular eczema
answer: B - erythema nodosum
- painful, erythematous, inflammatory nodules seen on anterior shins
- range in colors from pink, red, purple
- usually bilateral
- may also occur on other parts of body
- An idiopathic inflammatory skin condition; commonly associated with
⦁ ESTROGEN EXPOSURE - OCPs, Pregnancy
⦁ certain medications = SULFA related drugs, OCPs, estrogen
⦁ Infections: MC STREP, TB, Sarcoidosis, Coccidioidomycosis - fungal
⦁ IBD (crohns + UC)
⦁ leukemia
⦁ Behcets
Erythema nodosum is generally self-limiting and resolves spontaneously within a few weeks. Treat underlying cause
- can give NSAIDS for pain
- if persistent = give STEROIDS
A 33-year-old woman at 35 weeks’ gestation has constant pelvic pain. She says that she had a small amount of dark red vaginal bleeding after sexual intercourse. Her blood pressure is 80/50 mmHg, and she has tachycardia. The uterus is firm and tender to palpation. Which of the following is the most likely diagnosis?
(A) Abruptio placentae (B) Cervical laceration (C) Placenta previa (D) Preterm labor (E) Vasa previa
Answer: A - abruptio placentae
- DARK RED vaginal bleeding
- painful
- hypotension (80/50)
- tachycardia
Both vasa previa & placenta previa = lots of bright red blood
cervical laceration, is incorrect because this patient is in the antepartum stage of pregnancy, and cervical laceration typically results from passage of the fetus through the birth canal
preterm labor, is incorrect because there is no history of this patient having any uterine contractions, an obvious sign of preterm labor
vasa previa, is incorrect. This is a condition whereby the umbilical vessels overlay the internal cervical os and may be a cause of third-trimester bleeding. However, this patient does not have copious vaginal bleeding of bright red blood.
) A 41-year-old woman has a nine-month history of nausea, constipation, dyspepsia, general
fatigue, arthralgia, and increasing memory loss. She has no history of illness other than her
present complaints, and her menses have been regular. Physical examination shows no
abnormalities. Laboratory findings include the following levels:
Serum ⦁ Albumin 4.9 g/dL ⦁ Calcium 13.0 mg/dL ⦁ Chloride 111 mEq/L ⦁ Creatinine 1.0 mg/dL ⦁ Phosphate 0.3 mg/dL ⦁ Blood urea nitrogen 17 mg/dL
Which of the following is the most likely diagnosis?
(A) Cushing syndrome (B) Hyperparathyroidism (C) Hypopituitarism (D) Malabsorption (E) Multiple myeloma
Answer: B - Hyperparathyroidism
The patient has the classic signs of Hyperparathyroidism - which include gastrointestinal, musculoskeletal, and neurologic abnormalities. Additionally, the laboratory values clearly show hypercalcemia and hypophosphatemia, both of which indicate hyperparathyroidism.
Cushing syndrome, is incorrect because the patient does not have the typical
symptoms of Cushing syndrome, which include oligomenorrhea or amenorrhea and a host of
changes to the body habitus. Option (C), hypopituitarism, is incorrect because the patient does
not have severe metabolic, growth, and menstrual abnormalities as a result of a decrease in
pituitary hormone levels. Option (D), malabsorption, is incorrect because there is no indication in the patient’s history of weight loss, other illness, or abnormal menses. In addition, the laboratory values do not support a diagnosis of malabsorption.
Option (E), multiple myeloma, is incorrect because the patient does not have anemia, bone pain, proteinuria, and renal failure, which are characteristic of this disease. Although the patient does have hypercalcemia, which is another characteristic of multiple myeloma, none of the other presenting symptoms support this diagnosis. In addition, multiple myeloma is most frequently diagnosed in individuals aged 65 to 74 years, and the patient described is significantly younger.
A 40-year-old man is being examined because he feels weak and tired and has had symptoms of
sexual dysfunction. His blood pressure is 100/60 mmHg. Physical examination shows decreased
facial, axillary, and pubic hair. Laboratory studies show decreased serum levels of luteinizing and
follicle-stimulating hormones, thyroxine, and testosterone; serum thyroid-stimulating hormone
level is within normal limits. Which of the following is the most likely diagnosis?
(A) Diabetes insipidus (B) Diabetes mellitus (C) Hyperprolactinemia (D) Hypopituitarism (E) Hypothyroidism
Answer: D - Hypopituitarism
Option (A), diabetes insipidus, is incorrect because the patient does not have urinary symptoms or signs of dehydration, and the laboratory findings are not consistent with this condition.
Option (B), diabetes mellitus, is incorrect because diabetes mellitus is a metabolic disorder, specifically affecting carbohydrate metabolism. It is a disease characterized by persistent hyperglycemia. There is no mention of serum glucose findings in the history or laboratory studies of this patient.
Option (C), hyperprolactinemia, is incorrect because there is no evidence of increased serum
prolactin levels or galactorrhea in the patient. Option (E), hypothyroidism, is also incorrect.
Weakness and fatigue are symptoms of hypothyroidism, but the normal thyroid-stimulating
hormone level rules out a thyroid disorder in this patient.
The correct answer is Option (D), hypopituitarism,
based on the common symptoms of gonadotropin deficiency, weakness, fatigue, and sexual
dysfunction. The decreased levels of luteinizing and follicle-stimulating hormones indicate
hypogonadism. These laboratory findings, in addition to the decreased levels of testosterone,
indicate hypopituitarism.
A 22-year-old woman is brought to the emergency department for evaluation three hours after falling backward out of a chair and striking her head on a carpeted floor. She has tenderness over the back of her head but does not report loss of consciousness. Physical examination, including neurologic and musculoskeletal evaluation, shows no abnormalities except a 2-cm area of swelling and minimal abrasion overlying the left occiput. Which of the following is the most appropriate next step?
(A) Anteroposterior and lateral x-ray studies of the skull
(B) CT scan
(C) Discharge and observation by family or friends
(D) Hospital admission for observation
(E) MRI
Answer: (C), discharge and observation by family or friends.
Patients with minor head injuries can be discharged with observation instructions in the care of a reliable adult.
Option (B), CT scan, is incorrect because this patient does not meet the criteria for CT scan. These criteria include: instability after multiple traumas, unreliable history or examination because of possible alcohol use or drug ingestion, loss of consciousness for longer than five minutes, repeated vomiting or vomiting for more than eight hours after injury, post-traumatic seizures, progressive headache, physical signs of basilar skull fracture, or amnesia.
Option (D), hospital admission for observation, is incorrect because this patient does not meet
the criteria for hospitalization. These criteria include: coma, underlying pathology such as
coagulopathy and/or hydrocephalus, unreliable history or examination because of possible
alcohol use or drug ingestion, documented loss of consciousness for longer than five minutes,
severe and persistent headache, protracted vomiting, suspected child abuse, unreliable
caregiver, altered mental status or seizures, and focal neurologic deficit.
A previously healthy 15-month-old boy becomes anxious and begins crying and drooling
copiously. A few minutes earlier he had been calmly playing with his toys. Temperature is 36.7°C (98.1°F), pulse rate is 84/min, and respirations are 18/min. On physical examination, the posterior pharynx is mildly injected but otherwise clear. The lungs are clear to auscultation and percussion. Findings on chest x-ray study are normal. Within an hour he is calmer, but he
continues to drool heavily. Which of the following is the most appropriate next step?
(A) Administration of syrup of ipecac (B) Barium swallow x-ray study (C) Chest physiotherapy (D) Esophagogastroduodenoscopy (E) Insertion of a nasogastric tube
Answer: D - Esophagogastroduodenoscopy
This question tests the examinee’s ability to recognize foreign body ingestion and to determine
the appropriate next step in management and evaluation. The correct answer is Option (D),
esophagogastroduodenoscopy, which is the appropriate step to confirm a diagnosis and provide therapeutic treatment.
Option (A), administration of syrup of ipecac, is incorrect and may be dangerous. It is not
recommended that health care providers or parents attempt to dislodge a foreign body from a
spontaneously breathing patient by administering syrup of ipecac. Option (B), barium swallow
x-ray study, is incorrect because the barium may obscure the view of the foreign body. This
intervention is also contraindicated if esophageal perforation cannot be ruled out as a possible
diagnosis. Option (C), chest physiotherapy, is incorrect and not recommended as an intervention because it will most likely be ineffective or could potentially lead to aspiration of the foreign body. Option (E), insertion of a nasogastric tube, is also incorrect because this intervention does not have any diagnostic or therapeutic value in patients who ingest foreign bodies.
A mother brings her 6-day old infant to ED with fever & skin rash. fever began 2 days ago: reached 102. Irritable, numerous erythematous bullae across chest/abdomen/extremities. Nikolsky’s sign is present. No meds for infant
a) Pemphigus Vulgarus
b) SSSS - Staphylococcal scalded skin syndrome
c) Impetigo
d) Rubella
e) Erysipelas
Answer: SSSS
32 YO female complains of “acne” & persistent cough for last several months. She has tried OTC meds for her cough and acne, but both have gotten worse. Normal vital signs. PE = multiple flesh colored papules ranging from 2-8mm involving the alae nares, eyelids, forehead and rear of neck and hairline. Cervical lymphadenopathy. CXR = hilar adenopathy & pulmonary reticular opacities. What is appropriate treatment?
a) Albuterol
b) Prednisone
c) Dextromethorphan
d) Sulfasalazine
e) Topical Clindamycin/benzoyl peroxide
Answer: Prednisone
- Patient has sarcoidosis - Lupus Pernio = occurs in sarcoidosis
- patient also may have erythema nodosum
- albuterol if asthma; dextromethorphan = cough suppressant. Sulfasalazine = IBD & RA. Clinda/benzo if actually was acne
A 45-year-old man with history of previously well-controlled hypertension comes in because colleagues at his office are telling him that his appearance is changing. The patient has recently noted that his old shoes felt too tight and he had to buy new, larger shoes. He also complains of shooting pains in his hands which his company nurse told him were consistent with carpal tunnel syndrome. Review of systems reveals increased thirst and urination. His temperature is 37.0°C (98.6°F), blood pressure 190/90 mm Hg, pulse 73/min, and respirations 13/min. Physical examination shows frontal bossing, mandibular enlargement, and widening of the spaces between his teeth. His hands and feet seem larger and more out of proportion to his body size than usual.
Which of the following is the most appropriate diagnostic study at this time?
a) cortisol level
b) CT of brain
c) fasting glucose level
d) growth hormone
e) insulin-like growth factor
Answer = E: insulin-like growth factor
The correct answer is E. This patient has acromegaly, an excess of growth hormone, evidenced by increased hand and foot size and coarse facial features with a protruding jaw. Other features, such as tooth-space widening and macroglossia, can be present. Impaired glucose tolerance is often present, as is diabetes. Hypertension is present in 1/3 of these patients. Complaints of headache and visual field losses can occur. Patients may have carpal tunnel syndrome. The test of choice for diagnosis is insulin-like growth factor, which is elevated >5 times the normal limit. Another test which can be used for diagnosis is measurement of growth hormone after the administration of 75 g of glucose. In a normal patient, the growth hormone level would be suppressed, but in a patient with acromegaly, the levels remain high (greater than 5 ng/mL).
Growth hormone levels (choice D) should not be used as a screening test to diagnose acromegaly, because patients have a normal daily fluctuation of gonadotropin-releasing hormone (GNRH) and growth hormone. Growth hormone levels can also fluctuate with exercise and acute illness. Pregnant and adolescent patients can also have increased levels and not have acromegaly. A lack of the normal nadir may be useful in identifying patients who have acromegaly, because these patients will not have full suppression of growth hormone throughout the 24-hour period.
Cortisol levels (choice A) might be helpful as part of the diagnosis of Cushing syndrome. Cushing syndrome can be associated with insulin resistance, hypertension, and weight gain but doesn’t share other features associated with acromegaly.
A young mother brings her 6-month-old firstborn son for a health supervision visit. She states that the baby has been doing well and she has not noticed any abnormalities in his development, though her experience is really not very significant, she says. The boy is breastfeeding and is increasing the number of solid foods he eats with each passing day. He sleeps almost through the night and stopped having colic 2 months earlier. On physical examination the boy appears to be in no distress. He is in the 50th percentile for weight and length and his vital signs are normal. The health care provider performs a thorough neurologic exam in front of the mother, who is very curious about everything the health care provider is doing. On extension of the head, the boy flexes his arms and legs. When a finger is put in his palm, he flexes his hand, elbow and shoulder ipsilaterally. Tactile stimulation of the cheek prompts him to turn his mouth to that side, and touching the dorsum of his foot leads to a step-up movement. During a simulated fall, the baby reacts by extending his arms. The mother asks about each and every one of the newborn reflexes and when they are supposed to disappear. Which of the following is likely to be the one reflex that persists throughout life?
a) Extension of the head leads to flexion of the arms and legs
b) Finger in palm leads to flexion of the hand, elbow, and shoulder ipsilaterally
c) Stimulation of a fall leads to extension of arms
d) Stimulation of the cheek prompts turning of mouth to that side
e) Stimulation of the dorsum of the foot leads to a step-up movement
Answer: C - Stimulation of a fall leads to extension of arms
- the rest of the reflexes above disappear around age 4-6 months
A 40-year-old woman is brought to the emergency department following a suicide attempt with imipramine (Tofranil). Her fiancé found her unresponsive, with an empty bottle of the imipramine at her side. The imipramine had been his, and the prescription had been filled that morning. Her past medical history is significant for hypertension, atrial fibrillation, diabetes, and asthma. Her medications include furosemide, procainamide, glyburide, prednisone, and albuterol. She has no known drug allergies. She is afebrile, has a blood pressure of 100/60 mm Hg, pulse of 62/min, and respirations of 22/min. A gastric lavage yields multiple pill fragments. She is confused and somnolent, and has shallow respirations. Her physical examination is otherwise unremarkable. On an electrocardiogram, which of the following abnormalities would most likely reflect possible cardiac toxicity?
a) Left deviation of the QRS axis
b) Prolongation of the QT interval
c) Shortening of the PR interval
d) ST-segment depression
e) T-wave inversion
Answer: B - Prolongation of the QT interval
The correct answer is B. A prolongation of the QT interval is highly predictive of both cardiac and CNS toxicities from tricyclic antidepressant ingestion. This medication has high lethality associated with its overdose because of its cardiac effects.
A patient with systemic lupus erythematosus (SLE) very much wants to become pregnant. What should her health care provider tell her regarding pregnancy?
a) There is no increased risk to the fetus
b) There is an increased risk for cardiovascular malformations
c) There is an increased risk for nervous system malformations
d) There is an increased risk for renal malformations
e) There is an increased risk for spontaneous abortions and prematurity
answer: E - There is an increased risk for spontaneous abortions and prematurity
Patients who have SLE have an increased incidence of spontaneous abortion, fetal death in utero, and prematurity
A 54-year-old woman comes to the emergency department with severe left-sided flank pain that is referred to the left labia majora. She is very uncomfortable lying still on the stretcher and is continuously shifting, trying to find a comfortable position. A nonenhanced helical CT scan shows a 10-mm ureteral stone at the ureteropelvic junction. She has a normal coagulation profile. Which of the following would most likely be the best therapy in this case?
a) Plenty of fluids and analgesics and await spontaneous passage
b) Extracorporeal shock wave lithotripsy (ESWL)
c) Endoscopic retrograde basket extraction
d) Endoscopic retrograde laser vaporization of the stone
e) Open surgical removal
answer: B - Extracorporeal shock wave lithotripsy (ESWL)
Extracorporeal shock wave lithotripsy (ESWL) is the most commonly used method to fragment urinary stones and allow their passage. Pregnancy and coagulation problems are contraindications. The first one is ruled out by her age; the second one we have been told she does not have. Because the stone is 10 mm, it is less likely to pass on its own because of its large size.
Waiting for spontaneous passage (choice A) would have been perfect for a much smaller stone (3 mm) that had already negotiated most of the ureter. A 7-mm stone way up at the ureteropelvic junction has a very small chance of spontaneous passage.
Retrograde endoscopic approaches (choices C and D) are more invasive than ESWL. They would not be the first choice for this scenario.
Open surgical removal (choice E) would have been good for a much bigger stone. A huge target (3 cm or larger) could indeed be fragmented by ESWL, but then we would be contending with dozens of still very large stones. In those cases, a direct approach to extract the huge intact stone would work better.
A 49-year-old man is recuperating in a hospital after having sustained a recent cerebrovascular accident that damaged part of his right temporal lobe. His wife and children are at his bedside day and night. They want to actively participate in his rehabilitation but specifically ask the health care provider what they can expect from him in the future. Once the patient has recuperated from the immediate effects of his stroke, to which of the following psychiatric disorders will he be most predisposed?
a) Conversion disorder
b) Narcolepsy
c) Major depressive disorder
d) Schizophrenia
e) Substance abuse
answer: C - Major Depressive Disorder
The correct answer is C. Any event that affects the vasculature, such as a myocardial infarction or a cerebrovascular accident (CVA), has been shown to increase the risk of major depressive disorder in the months that follow. The pathophysiology of such a development is unclear, but is thought to be related to the effects of serotonin on vascular physiology.
A patient who is being treated for hypertension related to a myocardial infarction that occurred 2 hours ago is medicated with IV nitroprusside (Nipride). Which of the following is the expected action of this drug?
a) Constriction of arterioles alone
b) Constriction of both arterioles and venules
c) Constriction of venules alone
d) Dilatation of arterioles alone
e) Dilatation of arterioles and venules
answer: E - Dilatation of arterioles and venules
A 63-year-old woman comes to the emergency department with severe abdominal pain. She reports the pain is epigastric and worse with eating. There is some radiation to the back. There is associated diarrhea that is described as “fatty.” Symptoms have been present for 3 to 4 months. Her medical history is significant for 2 episodes of acute pancreatitis, hypertension, and hyperlipidemia, but she takes no medications for these disorders. Her temperature is 38.3ºC (100.9ºF), blood pressure 160/90 mm Hg, pulse 143/min, and respirations 16/min. Physical examination shows epigastric tenderness, mild distention, and decreased bowel sounds. She appears uncomfortable and in moderate distress. An obstruction series reveals only scattered pancreatic calcifications. Laboratory studies show:
Amylase 45 U/L (normal 23-140 U/L) Lipase 10 U/L (normal <160 U/L) Albumin 2.8 g/dL (normal 3.4-5.4 g/dL) AST 98 IU/L (normal <40 IU/L) ALT 45 IU/L (normal <40 IU/L) Total bilirubin 1.1 mg/dL (normal <1.9 mg/dL)
Which of the following is the most likely underlying etiology for this patient’s symptoms?
a) Alcohol
b) Elevated triglycerides
c) Gallstones
d) Hypercalcemia
e) Idiopathic
answer: A - Alcohol
This patient has chronic pancreatitis. The finding of calcifications in the pancreas is a good clue to this patient’s diagnosis. Amylase and lipase can be normal or elevated, depending on how much intact pancreas is left. Seventy percent of chronic pancreatitis is caused by alcohol ingestion. Our patient’s malnutrition, as evidenced by her low albumin and her AST to ALT ratio of 2:1, is consistent with alcoholic hepatitis. The treatment is pain relief, a low-fat diet with medium chain triglycerides, vitamins, and pancreatic enzyme replacement. The remaining 30% of chronic pancreatitis is idiopathic (choice E).
- **AST : ALT > 2 = ALCOHOL HEPATITIS
- **ALT > AST = think LIVER: viral / toxic / inflammatory processes
- **ALT > 1000 = autoimmune hepatitis
Elevated triglycerides (choice B) are a rare cause of acute pancreatitis, not chronic pancreatitis. Triglyceride levels in these patients are usually >1,000 mg/dL. Gallstones (choice C) are an important cause of acute pancreatitis but are not typically associated with chronic disease. Patients who have gallstone pancreatitis are usually overweight, fertile, and in their forties. Correction is by ERCP removal or laparoscopic cholecystectomy with common duct exploration and removal of the choledocholithiasis. Hypercalcemia (choice D) is a rare cause of acute pancreatitis. Pancreatic calcifications seen on obstruction series do not represent elevated serum calcium levels. Hypocalcemia can complicate acute pancreatitis as a result of saponification of fats, which takes place due to pancreatic insufficiency because of acute pancreatitis.
A 20-year-old woman comes to the health care provider because of left lower quadrant pain for 2 months. She states that she first noticed the pain 2 months ago but now it seems to be getting worse. She has had no changes in bowel or bladder function. She has no fever or chills and no nausea, vomiting, or diarrhea. The pain is intermittent and sometimes feels like a dull pressure. Pelvic examination is significant for a left adnexal mass that is mildly tender. Urine hCG is negative. Pelvic ultrasound shows a 7-cm complex left adnexal mass with features consistent with a benign cystic teratoma. Which of the following is the most appropriate next step in management?
a) Repeat pelvic examination in 1 year
b) Repeat pelvic ultrasound in 6 weeks
c) Prescribe the oral contraceptive pill
d) Perform hysteroscopy
e) Perform laparotomy
Answer: E - Perform Laparotomy
To repeat pelvic examination in 1 year (choice A) would not be correct management. This patient is symptomatic with a 6-cm ovarian mass that is at risk for torsion. She therefore should be managed surgically.
To repeat pelvic ultrasound in 6 weeks (choice B) is appropriate for some adnexal masses. For example, in a young woman who has a small complex cyst that appears consistent with a corpus luteum, it may be most prudent to recheck an ultrasound in 6 weeks to see if the cyst has resolved. This patient, however, is symptomatic with a 6-cm cyst that appears to be a dermoid, which will not resolve spontaneously. She therefore requires surgery.
To prescribe the oral contraceptive pill (choice C) may help to prevent future ovarian cysts, but it will not resolve this cyst, which requires surgical management.
To perform hysteroscopy (choice D) would not be indicated. Hysteroscopy is used to evaluate the uterine cavity and fallopian tubes and would not be used for management of an adnexal mass.
A 30-year-old woman with a history of tonic-clonic seizures complains of double vision, thickened gums, and growth of facial hair since starting a new medication. Which of the following anticonvulsant medications is most likely responsible for her symptoms?
a) Carbamazepine (Tegretol)
b) Ethosuximide (Zarontin)
c) Phenobarbital (Phenobarb)
d) Phenytoin (Dilantin)
e) Valproic acid (Depakene)
Answer: D - Phenytoin
A 45-year-old male comes to the emergency room after being involved in a head-on motor vehicle accident earlier in the day. The patient notes that he struck his head, but he did not experience any loss of consciousness. His blood pressure is 190/110, his respirations are irregular, and his electrocardiogram (EKG) shows sinus bradycardia with a heart rate of 42 beats per minute. The patient’s symptoms are part of which clinical triad?
Beck’s triad
Charcot’s triad
Bergman’s triad
Cushing’s triad
Answer: D - Cushing’s triad
- A change in respirations, often irregular and deep, such as cheyne stokes.
- A widening pulse pressure (the difference between the Systolic and the Diastolic BP
- Bradycardia (slow heart rate).
- indicates increased ICP
All of the following are minor manifestations of acute rheumatic fever as described by the modified Jones criteria EXCEPT
Erythema marginatum
Leukocytosis
Elevated erythrocyte sedimentation rate (ESR)
arthralgia
Answer: A - Erythema Marginatum
Jones Criteria (JONES acronym = major criteria)
⦁ Joint (migratory polyarthrtitis) - 2 or more joints affected; medium/large = MC
⦁ Oh my heart (carditis) - mitral/aortic, myocarditis, pericarditis
⦁ Nodules (subcutaneous nodules) - rare - seen over joints, scalp & spinal column
⦁ Erythema Marginatum - non-itchy annular rash with round sharp borders on trunk/extremities, NOT on face
⦁ Sydenham’s Chorea
Jones Criteria (minor criteria) ⦁ fever, arthralgia, increased ESR/CRP/leukocytosis
A 19-year-old woman comes to the office complaining of a painful rash on her elbows and knees. The rash appears as raised erythematous areas topped with silvery, scaling skin. She reports, “The rash is very itchy.” She had similar symptoms several weeks before, but they spontaneously resolved without treatment. Which of the following is most likely to be the diagnosis?
a) Impetigo
b) Tinea corporis
c) Rosacea
d) Psoriasis
Answer: D - Psoriasis
During a colonoscopy, the gastroenterologist notices that the patient’s colon wall has a “cobblestone” appearance. Which of the following is the most likely diagnosis?
a) Crohn’s disease
b) Celiac sprue
c) Ulcerative colitis
d) Whipple’s disease
Answer: A - Crohns
You are evaluating an obese 37-year-old female in the ER. She has been complaining of right-sided abdominal pain and excessive flatulence. She normally has the pain after eating, but it usually resolves on its own. This episode has persisted for several hours. On physical examination, you palpate her right-upper quadrant while she takes a deep inspiration. Discomfort during this maneuver is referred to as a positive:
a) Levine’s sign
b) Brudziński’s sign
c) Psoas sign
d) Murphy’s sign
Answer: Murphy’s Sign
Murphy’s sign = A positive Murphy’s sign is seen in acute cholecystitis. It is elicited by firmly placing a hand at the costal margin in the right upper abdominal quadrant and asking the patient to breathe deeply. If the gallbladder is inflamed, the patient will experience pain and catch their breath as the gallbladder descends and contacts the palpating hand.
Brudzinski’s sign = to test meningeal irritation - flexion of the neck causes flexion at the hip and knees
You are acting as the first assist in the operating room, and the surgeon asks you to close an abdominal incision with an absorbable suture material. Based on the following choices, which suture would be your pick?
a) Silk
b) Vicryl
c) Dermabond
d) Nylon
Answer: B - Vicryl
All of the following are symptoms of esophageal achalasia EXCEPT
Dysphagia
Acid reflux
Hematochezia
Chest pain
answer: C - Hematochezia
Which of the following is NOT part of CREST syndrome? Calcinosis Sclerodactyly Esophageal dysmotility Solar urticaria
answer: d: solar urticaria
A 20-year-old female recently diagnosed with chlamydia comes to your office for swelling and pain in her knees bilaterally. The most likely diagnosis for this woman’s complaints is
Reiter’s syndrome
Sjögren’s syndrome
Down syndrome
Turner syndrome
answer: A - Reiter’s syndrome
- AUTOIMMUNE RESPONSE to an INFECTION in another part of the body
1) ARTHRITIS - asymmetric inflammation
2) CONJUNCTIVITIS / UVEITIS
3) URETHRITIS / CERVICITIS
A 26-year-old female comes to the ER with complaints of white vaginal discharge and pelvic pain. She admits to having unprotected sex. On physical examination, she has an inflamed cervix and cervical motion tenderness. Which one of the following two-medication pairs should she receive prior to leaving the ER?
Ceftriaxone 250 mg IM and clindamycin 300 mg po
Clindamycin 300 mg po and azithromycin 1 gm po
Ceftriaxone 250 mg IM and azithromycin 1 gm po
Mefoxitin 2 gm IV and azithromycin 1 gm po
Ceftriaxone 250 mg IM and azithromycin 1 gm po
Which of the following cardiovascular disorders is most likely associated with abnormal pupillary responses (Argyll Robertson pupil)?
a. Aortic regurgitation
b. Coarctation of the aorta
c. Thoracic aortic aneurysm
d. Myocarditis
Answer: C - The Argyll Robertson pupil (a pupil that constricts with accommodation but not in response to light) is characteristic of central nervous system syphilis that often results in thoracic aortic aneurysm.
Which of the following cardiovascular disorders is most likely associated with jugular venous distension and exophthalmoses?
A. Massive tricuspid regurgitation
B. Coarctation of the aorta
C. Thoracic aortic aneurysm
D. Myocarditis
Answer: A - Massive tricuspid regurgitation produces a markedly elevated venous pressure, usually manifested by a severely engorged (often pulsating) liver. If the venous pressure is sufficiently elevated, exophthalmoses may result.
Flushing and paling of the nail beds (Quincke pulse) and a bounding radial pulse are typical physical findings found in which of the following cardiovascular disorders?
A. Tricuspid regurgitation
B. Coarctation of the aorta
C. Thoracic aortic aneurysm
D. Aortic regurgitation
Answer: D - Aortic Regurgitation
Quincke pulse, which consists of alternate flushing and paling of the skin or nail beds, is associated with aortic regurgitation. Other characteristic features of the peripheral pulse in aortic regurgitation include the waterhammer pulse and pulsus bisferiens.
Conjunctivitis, urethral discharge, and arthralgia are common symptoms present in which of the following cardiovascular disease?
A. Tricuspid regurgitation
B. Coarctation of the aorta
C. Myocarditis
D. Aortic regurgitation
Answer: C - Myocarditis
Conjunctivitis, urethral discharge, and arthralgia are common symptoms present in myocarditis i.e. inflammation of myocardium.
A patient arrives with short stature, webbed neck, low-set ears, and epicanthal fold. Which of the following cardiovascular disorders he is most likely suffering from?
A. Tricuspid regurgitation
B. Coarctation of the aorta
C. Myocarditis
D. Aortic regurgitation
Answer: B - Short stature, webbed neck, low-set ears, and epicanthal folds are common clinical findings found in coarctation of the aorta which is a narrowing of part of the aorta (the major artery leading out of the heart). It is a type of birth defect.
Which of the following disorders are usually characterized by weight loss, depressed mood, feeling of tired, fatigue and frequent, nonspecific abdominal pain?
A. Pheochromocytoma
B. Pancreatic carcinoma
C. Adrenocortical insufficiency
D. Cushing syndrome
Answer: B - Pancreatic carcinoma should always be considered in depressed middle-aged patients. It’s symptoms include weight loss, abdominal pain, apathy, decreased energy, lethargy, anhedonia, and depression.
Release of which of the following hormones is inhibited by insulin?
A. Glucagon
B. ADH
C. Beta cells
D. Somatostatin
Answer: A - Glucagon
Increases in insulin levels inhibit glucagon release from α-cells of the pancreas. This paracrine effect is a basic element in insulin’s control of glucagon’s activation of both hepatic gluconeogenesis and lipolysis in adipose tissue.
Which of the following metabolic effects is caused by insulin release?
A. Increased breakdown of fats
B. Increased breakdown of proteins
C. Decreased blood sugar
D. Causes glucose to be phosphorylated in kidney
Answer: C - One of the key metabolic effects of insulin is to reduce blood glucose (sugar) levels. The deficiency of insulin or resistance to its effect, thus, leads to diabetes mellitus.
All of the following statements are true about Type I diabetes mellitus EXCEPT:
A. It may be linked to autoimmune issues
B. Onset is usually prior to age 20
C. Beta islet cells are destroyed
D. Insulin injections are unnecessary
D. Insulin injections are unnecessary
Answer: D - As Type 1 diabetes mellitus is caused by destruction of Beta islet cells, a deficiency of insulin results. Therefore, treatment plans involve giving insulin injections to such patients to help normalize their insulin levels.
Which of the following is NOT a complication of chronic diabetes mellitus?
A. Atherosclerosis
B. Neuropathy
C. Glaucoma
D. Hypotension
Answer: D - Hypertension, not hypotension, is considered as one of the common complications of long-term diabetes mellitus.
A 25-year-old African American woman comes to the clinic because of lesions on her legs. History reveals the lesions are painful, red nodules that have been present for the past five days. She also reports increased fatigue, unintentional weight loss, painful joints and blurry vision. Physical examination reveals a well-developed female with multiple erythematous macules and nodules on her shins. Which is the best initial test to detect the most likely diagnosis?
Abdominal CT Biopsy of the rash Chest radiograph Electrolytes and serum markers Peripheral blood smear and CBC with differential
Answer: B - biopsy the rash
To diagnose the patient with sarcoidosis, a biopsy of a skin lesion or other manifesting granuloma is needed. A biopsy would show non-caseating granulomas, a hallmark of the disease. The skin lesion described is classic for erythema nodosum which typically affects the shins
A 37-year-old African-American woman comes to the clinic because of fever, anorexia, and shortness of breath. She states that she has these symptoms from time to time, but this time is the worst and has lasted 3 days. Temperature is 36.6°C (97.9°F), pulse is 68/min, respirations are 16/min, and blood pressure is 128/78 mm Hg. Physical examination shows red/purple raised nodules on her shins, anterior uveitis, and an asymmetric smile. Which of the following findings is necessary for the most likely diagnosis of this condition?
Arthropathy Bilateral hilar adenopathy Increased serum levels of ACE Myopathy Non-caseating granulomas
Answer: E - Non-caseating granulomas
Sarcoidosis is a chronic systemic granulomatous disease often involving multiple organ systems. It occurs most often in female African-Americans. Features include constitutional symptoms, erythema nodosum, dyspnea on exertion, anterior uveitis, and Bell’s palsy, amongst others. Non-caseating granulomas are required for the diagnosis of the disease. Arthralgias and myopathies can be present. Bilateral hilar adenopathy is a common finding of the disease and is important in mobidity and mortality. Cardiac involvement is also seen in about 5% of cases, while Increased serum levels of ACE (choice D) are seen in 50% of cases. Treatment of sarcoidosis is generally with a steroid regimen.
A 35-year-old African American woman comes to the clinic complaining of vision problems, fatigue, and constipation. She reports a history of kidney stones and occasional rash. Her temperature is 37.1°C (98.8°F), pulse is 76/min, respirations are 16/min, and blood pressure is 120/85 mm Hg. Physical examination shows tender red nodules as displayed below. Which of the following enzymes, consistent with her other findings, is most likely over-expressed?
Alpha-1 Hydroxylase Adenylate cyclase Alkaline phosphatase Desmolase Sodium-potassium ATPase
Alpha-1 Hydroxylase
A 25-year-old African American man comes to the clinic because of cough. He also notices that he’s been becoming short of breath on his daily runs over the last few weeks. He denies any chest pain or wheezing. His temperature is 36.6 °C (97.9°F), pulse is 68/min, respirations are 16/min, and blood pressure is 128/84 mm Hg. Physical examination shows his lungs are clear to auscultation bilaterally. A chest x-ray is obtained and shown below. Which of the following serum laboratory values is most likely to be abnormal?
1,25-dihydroxyvitamin D Adenosine deaminase Angiotensin-converting enzyme Calcium Quantiferon gold
Answer: ACE
calcium is elevated in sarcoidosis, but ACE is elevated in 60-80% of patients with sarcoidosis, hypercalcemia is only found in 10-15% of patients
A 20-year-old man comes to the clinic because of shortness of breath on exertion. He says he has felt short of breath for three weeks and also had a rash occasionally. His temperature is 36.6 °C (97.9°F), pulse is 68/min, respirations are 18/min, and blood pressure is 130/78 mm Hg. Physical examination shows a raised erythematous macular rash. A chest x-ray shows bilateral hilar lymphadenopathy and his serum angiotensin converting enzyme is elevated. A biopsy of his rash shows multiple non-caseating granulomas with multinucleated giant cells. Which of the following is most likely to be seen on neurological examination?
Facial nerve palsy Hypothalamic dysfunction Cerebellar ataxia Peripheral neuropathy Papilledema
Facial nerve palsy
A 25-year-old African American man comes to the clinic because of cough for 3 months and intermittent fevers. He also notices that he’s been becoming short of breath on his daily runs over the last few weeks. He denies any chest pain or wheezing. His temperature is 36.6 °C (97.9°F), pulse is 64/min, respirations are 16/min, and blood pressure is 128/76 mm Hg. Physical examination shows an oxygen saturation of 94%. His lungs are clear to auscultation. Chest computerized tomography shows bilateral mediastinal and hilar adenopathy as well as well-defined bilateral nodules. The pathology from the biopsy of a nodule is shown. Which of the following is the most appropriate treatment for this condition?
Albuterol Chemotherapy Itraconazole Prednisone RIPE
Prednisone
A 52-year-old male presents complaining of urinary frequency, with hesitancy, and nocturia for the past few months. During his physical examination, you note a nontender, non-enlarged prostate with an isolated right posterior lobe nodule. Which of the following options is most appropriate?
A. Order a serum acid phosphatase level
B. Initiate prazosin and schedule a follow-up appointment in 6 weeks
C. Refer the patient for an ultrasound of the prostate and order a PSA level
D. Reassure the patient and schedule a follow-up appointment in six months
E. Initiate norfloxacin therapy for 7 days and schedule follow-up in two weeks
Answer: C
Refer the patient for an ultrasound of the prostate and order a PSA level
This patient has an isolated nodule of the prostate gland — cancer until proven otherwise. You should order an ultrasound and a PSA. BPH will present as a diffuse enlargement, and not a discrete nodule.
Which term is used to describe the characteristic concave or “spoon-shaped” nails of iron deficiency anemia?
A. leukonychia B. koilonychias C. clubbing D. onycholysis E. paronychia
Answer: B
Koilonychia
Koilonychia is a spoon-shaping of the nail itself. It is usually a result of iron deficiency anemia. Leukonychia is often associated with hypoalbuminemia that causes partial or complete white discoloration of the nails. Leukonychia may also appear as a rare side effect of systemic chemotherapy in some oncological patients but may also be present with arsenic poisoning, renal failure pneumonia, or heart disease. Clubbing of the nails is an actual thickening or elevation of the nail bed - it is a sign of a release of TNF associated with pulmonary disorders (tissue necrosis factor) typically found in bronchiectasis, lung cancers and cystic fibrosis (the nails are NOT necessarily cyanotic.) Onycholysis is a painless separation of the nail from the nail bed. Several or all nails are usually affected - there are many causes. Paronychia is an infection of the nail bed and nail margin, usually from trauma or more commonly, nail-biting.
Which of the following is NOT a characteristic feature of nephrotic syndrome?
A. proteinuria B. hematuria C. hypoalbuminemia D. hyperlipidemia E. generalized edema
Answer: B
Hematuria
Hematuria is present in NEPHRITIC syndrome. Nephrotic syndrome characteristically includes proteinuria (>3.5 gm/day), with resulting low serum albumin, hyperlipidemia, hypertension, hypercoagulability, and generalized edema (from oncotic third-spacing)
A patient describes a history of recurrent bouts of uveitis. Her chemistry panel reveals elevated serum calcium and uric acid levels. Her anergy screen is negative. Her chest x-ray demonstrates bilateral hilar adenopathy. Which diagnosis is most likely?
A. Silicosis B. Sarcoidosis C. Alpha-1 antitrypsin deficiency D. Histoplasmosis E. Tuberculosis
Answer: B
Sarcoidosis
Sarcoidosis typically presents with hilar lymphadenopathy and noncaseating granulomas of the lungs (and other organs). In addition, patients may have eye involvement (uveitis). Elevations of ACE, Calcium and uric acid are frequently seen.
A 34-year-old female presents complaining of symmetrical redness and swelling of the small joints of her hands (PIPs and MCPs). She has noted that the symptoms are worst in the morning. Her erythrocyte sedimentation rate is elevated and her rheumatoid factor is negative. Which of the following diagnosis is most likely?
A. progressive systemic sclerosis B. CREST syndrome C. osteoarthritis D. rheumatoid arthritis E. ankylosing spondylitis
Answer: D
Rheumatoid Arthritis
In spite of the negative RF, Rheumatoid arthritis is the most likely diagnosis. RA characteristically includes small joint symmetrical arthritis, with an elevated ESR (therefore inflammatory, and not OA). 80% of patients with RA will have a positive RF, but 20% will be negative. PSS involves squamous cell thickening and sclerosis causing taut skin of the face and hands and difficulty with esophageal motility. CREST syndrome is a subset of PSS; Ankylosing spondylitis would have an elevated ESR and negative RF, but mainly involves the SI joint and lumbar/thoracic spine fusion (bamboo spine)
A 27-year-old nulliparous female presents because she’s been trying to get pregnant for two years, but has failed. She relates a history of a misdiagnosis of appendicitis that lead to abscess formation when she was 14 years old. Which of the following diagnostic studies would be most helpful at this point in her evaluation?
A. TSH level B. hysterosalpingogram C. laparoscopy D. PAP smear E. pelvic ultrasound
Answer: B
Hysterosalpingogram
While I would disagree that an invasive procedure like HSG should be done first-line, the thing to remember in this question is that the patient has reason to have tubal scarring from adhesions (and there is no better answer listed to choose), so, for a board exam I would choose this answer. The TSH level would not be indicated (she has not had a pg loss), lap could diagnose the tubal scarring but would be done after an abnormal HSG. Pap smear is screening for cervical cancer and not indicated in this case of infertility, and the pelvic US would yield nothing diagnostically about the tubes.
A 14-year-old is experiencing a severe asthma attack. Although he is using accessory muscles to breath, auscultation of his chest reveals no audible wheezing. His heart rate is 160 and his respiratory rate is 52. Which of the following arterial blood gases represents the worst prognosis?
A. pH = 7.52; pC02 = 28; p02 = 80 B. pH = 7.44; pC02 = 38; p02 = 70 C. pH = 7.60; pC02 = 18; p02 = 60 D. pH= 7.40; pC02 = 40; p02 = 60 E. pH = 7.27; pC02 - 62; p02 = 64
Answer: E
pH = 7.27; pC02 - 62; p02 = 64
This patient has a respiratory rate of 52. If she is ventilating, she is blowing off C02 (an acid) and would be alkalotic and should have a low C02. A pH which is acidic with a pC02 which is elevated means that she is no longer ventilating at all (she needs mechanical ventilation or she will die).
While suturing a wound, you opt to use lidocaine with epinephrine. The rationale for your choice is:
A. an increase in the absorption of the lidocaine.
B. an increase in the diffusion of the lidocaine into the nerve’s myelin sheath.
C. an increase in the blood flow to the area of injection.
D. an increase in the duration of anesthesia.
E. a decrease in the risk of infection at the site of injection.
Answer: D
An increase in the duration of anesthesia
Lidocaine with epinephrine both increases the duration of anesthesia and decreases blood flow to the area of injection-i.e. penis, nose, fingers, toes
An EKG demonstrates a PR interval of 0.16 seconds, a P to QRS relationship of 1:1, a variable heart rate and an R to R interval that is noted to accelerate ad decelerate during the respiratory cycle. What is the diagnosis?
A. Wenckebach B. third degree heart block C. atrial fibrillation D. sinus arrhythmia E. atrial flutter
Answer: D
sinus arrhythmia
This is sinus arrhythmia. Wenckebach & third-degree AVB would have a nonconducted P wave. A-fib would have no P waves and an irregularly irregular rhythm. A flutter would have “flutter waves” or a regular rhythm of 150.
A 42-year-old female presents after finding a firm, painless bump in her right eyelid. On examination, you note a 6 mm mass within the tarsus of the right eye. The skin is freely movable over the mass. The remainder of the ophthalmoscopic examination is unremarkable. Which of the following is the most likely diagnosis?
A. Pterygium B. Chalazion C. Ectropion D. External hordeolum E. Internal hordeolum
Answer: B
Chalazion
A chalazion is a painless chronic mass in the eyelid. Hordeolum are acute and red and painful. Pterygium involves the sclera. Ectropion is when the eyelid sags outwardly and the lid doesn’t close well.
A 22-year-old patient was involved in an automobile accident and is comatose. Which of the following diagnostic modalities would be least useful in this patient’s evaluation?
A. CT scan B. skull radiographs C. MRI scan D. EEG E. PET scan
Answer: B
Skull radiographs
Of these choices, the skull film is LEAST useful. Whether or not the skull is fractured, any LOC in a head trauma requires imaging for bleeding. CT would be the imaging test of choice. MRI should be done for a more chronic bleed. EEG would help to establish brain activity. PET scan (while not done often) would establish physiologic function (uptake of glucose). A positive or negative plain film of the skull would supply NO FURTHER INFORMATION
Which white blood cell disorder is characterized by the presence of the Philadelphia chromosome in 90% of cases?
A. chronic lymphocytic leukemia (CLL) B. acute lymphocytic leukemia (ALL) C. chronic myelogenous leukemia (CML) D. acute myelogenous leukemia (AML) E. multiple myeloma
Answer: C
Chronic Myeloid Leukemia (CML) - (Philadelphia CreaM cheese)
Philadelphia Chromosome occurs in CML. ALL occurs in children. AML is associated with Auer rods. Multiple myeloma has Bence-Jones protein. CLL has no clear distinguishing feature except increased lymphocytes.
Which of the following thyroid profiles is most compatible with a diagnosis of primary hypothyroidism?
A. a low TSH (thyroid stimulating hormone) level and a high T4 B. a low TSH level and a normal T4 C. a low TSH level and a low T4 D. a high TSH level and a low T4 E. a high TSH and a high T4
Answer: D
High TSH level and a low T4
Low T4 is diagnostic for low thyroid function. If the pituitary is normal (as in primary thyroid disease) the TSH should be high as the pituitary tries to stimulate the failing thyroid gland.
All of the following are factors that predispose a patient to the development of gastroesophageal reflux EXCEPT:
A. hiatal hernia B. pregnancy C. scleroderma D. an incompetent esophageal sphincter E. pernicious anemia
Answer: E
Pernicious anemia
Pernicious anemia has no correlation with GERD. It is an autoimmune destruction of the gastric parietal cells that make intrinsic factor. Signs and symptoms are not present until B12 levels are very low (and include peripheral neuropathies & ataxia)
A patient warrants antihypertensive medication use for the duration of her pregnancy. Which of the following is the antihypertensive recommended for such patients?
A. alpha-methyldopa B. captopril C. nifedipine D. propranolol E. clonidine
Answer: A
Alpha-methyldopa
Methyldopa (Aldomet) is indicated in pregnancy. ACEI’s (captopril) are contraindicated in pregnancy. ß Blockers can be used but may cause growth restriction. Diuretics are not used in pregnancy. Calcium channel blockers and centrally acting agents (while not contraindicated) should be avoided.
A 6-year-old child falls onto his right arm. An x-ray demonstrates a buckle in the cortices of the distal radius, proximal to the growth plate, without angulation. What is the term used to describe this fracture?
A. Salter-Harris Type III B. Salter Harris Type IV C. Salter Harris Type V D. torus E. greenstick
Answer: D
Torus fracture
Torus or buckle fracture is most common in a child. This is proximal to the epiphyseal plate and so is not a Salter-Harris issue. Greenstick fracture is also common in children in long bones and is a fracture which “bends” the bone without fracturing it.
A 55-year-old male presents complaining of “difficulty wrtiing” using his dominant hand and some “slurred” speech. He has a h/o hypertension, DM type II, and hypertriglyceridemia. Which of the following would you anticipate to find on a CT scan of his head, as the explanation for his chief complaint?
A. hemorrhage in the distribution of his posterior cerebral artery
B. hemorrhage in the distribution of the middle cerebral artery
C. hypodensity measuring 12 mm by 21 mm in the distribution of the posterior cerebral artery
D. hypodensity measuring 4 mm by 4 mm in the internal capsule
E. calcifications bilaterally, in the third ventricles
Answer D
Hypodensity measuring 4 mm by 4 mm in the internal capsule
The main clue to this question is that most strokes are ischemic - especially with this patient’s history (ruling out hemorrhagic infarct - A & B). Calcifications (E) are not indicative of stroke at all. When trying to decide between C & D - the size of the area needs to correlate with the patient’s symptoms. If the patient had a HUGE infarct (12 x 21) he would have significantly more signs and symptoms than just some slurred speech and difficulty writing. (In addition, as it turns out, the internal capsule is supplied by the MCA - which is the most common vessel involved in an ischemic stroke)
A female in her third trimester of pregnancy developed hypertension, diffuse edema, proteinuria and hyperreflexia. She was treated with intravenous magnesium sulfate and is now hyporeflexic and drowsy. What do you prescribe now?
A. calcium B. diazepam C. an amphetamine D. additional magnesium E. oxygen
Answer: A
Calcium (Calcium Gluconate more specifically)
Pre-eclampsia causes hyperreflexia. Magnesium is the treatment. The sign of magnesium toxicity is a loss of reflexes… this is treated with calcium
This papulosquamous eruption is most common in young adults. A single oval patch is generally noted several days before a more generalized, fawn-colored rash erupts. This rash is most prevalent on the trunk, and the proximal upper and lower extremities. The rash spontaneously disappears over five to six weeks. The diagnosis is:
A. pityriasis rosea B. tinea corporis C. psoriasis D. atopic dermatitis E. sporotrichosis
Answer: A
Pityriasis rosea
This is the classic presentation of pityriasis rosea. Remember that the differential diagnosis includes secondary syphilis, so if there is a positive sexual history - consider ordering a VDRL/RPR. The “herald” patch which is described is pathognomonic for pityriasis.
A patient presents complaining of generalized swelling. Her urine is positive for protein. Her serum testing reveals hyperlipidemia and hypoalbuminemia. Which of the following is the most likely diagnosis?
A. cirrhosis B. nephrotic syndrome C. congestive heart failure D. cystitis E. pyelonephritis
Answer: B
Nephrotic syndrome
This is a classic presentation of nephrotic syndrome.
- hyperlipidemia
- proteinuria
- hypoalbuminemia
- generalized edema
A patient is experiencing an acute exacerbation of asthma. Which of the following drugs would be the least useful in the management of this asthma attack?
A. albuterol B. cromolyn sodium C. prednisone D. theophyline E. epinephrine
Answer: B
Cromolyn sodium
Cromolyn is a mast cell stabilizer and must be present in the system PRIOR to the symptoms. It would be useless in a patient already having symptoms.
A 34-year-old female patient presents to the ER with sharp pleuritic-type chest pain. An x-ray shows an enlarged cardiac silhouette suggestive of pericarditis. On more careful questioning, the patient admits to a polyarticular arthritis involving the small joints of her hands. Her obstetrical history is positive for two third trimester spontaneous abortions. At this point, you would suspect which autoimmune process as the cause of her symptoms:
A. Rheumatoid arthritis B. Systemic Lupus Erythematosis C. Wegener's granulomatosis D. Sjogren's Syndrome E. Scleroderma
Answer: B
Systemic Lupus Erythematosus
40-60% of patients with SLE have serositis (pleuritis and pericarditis). That, combined with joint pains, habitual AB - should cause you to order an ANA.
A patient with AIDS develops severe headaches. A CT scan demonstrates multiple ring-enhancing lesions of the brain. What diagnosis is most likely?
A. Toxoplasmosis B. Histoplasmosis C. lymphoma D. Cytomegalovirus E. Herpes encephalitis
Answer: A
Toxoplasmosis
Toxoplasmosis occurs commonly in HIV and is a parasitic infection causing ring-enhancing lesions (but lots of things can cause ring-enhancing lesions). CMV is common as well and typically causes retinitis. Herpes encephalitis can happen frequently too but causes a diffuse encephalitis.
A Caucasian male has a history of a meconium ileus as a newborn, steatorrhea and a positive sweat test. What is the most likely diagnosis?
A. Von Gierke’s disease B. Hurler’s syndrome C. Cystic fibrosis D. Hunter’s syndrome E. Cri du chat syndrome
Answer: C
Cystic fibrosis
This is the classic presentation of Cystic fibrosis
A 32-year-old male presents with dyspnea and a nonproductive cough. He is tachycardic, tachypneic, and febrile. Auscultation of his chest reveals scattered rhonchi. His chest x-ray demonstrates a diffuse interstitial infiltrate. His ABG demonstrates moderate hypoxemia and his LDH is elevated. What is the most likely diagnosis?
A. Streptococcal pneumoniae pneumonia B. Mycoplasma pneumoniae pneumonia C. Pneumocystis carinii pneumonia D. Bowen's disease E. Steven-Johnson syndrome
Answer: C
Pneumocystis carinii pneumonia
Formerly PCP Pneumonia now called (PJP) Pneumocystis jiroveci. Many AIDS patients present initially with PJP. These are the classic signs and symptoms of PJP pneumonia. While mycoplasma patients can be acutely ill, they are more likely (especially on the Boards) to present as mildly ill with a normal physical exam and no hypoxia. S. pneumonia is the most common community-acquired pneumonia but would present with a focal/lobar consolidation.
Common in HIV-infected patients with a low CD4 count of less than 200
Prophylaxis for high-risk patients with a CD4 count of less than 200 or with a history of PJP infection
A patient describes a history of a “curtain being brought down over my right eye it stayed there for a few minutes then was lifted back up.” In which of the following arteries is the etiology of this complaint located?
A. anterior cerebral B. middle cerebral C. posterior cerebral D. internal carotid E. external carotid
Answer: D
Internal Carotid
This is amaurosis fugax….a form of “TIA” of the optic artery — it arises from the internal carotid - most common location for occlusion in TIA.
Upon review of an EKG you notice periodic, repetitively absent QRS complexes and “grouped beating.” Which of the following is the explanation?
A. Third degree heart block B. Mobitz Type II heart block C. Mobitz Type I heart block D. first degree heart block E. sinus tachycardia
Answer: C
Mobitz Type I heart block
I don’t like the wording of this question, but typically “group beating” suggests Wenckebach. The fact that there are absent QRS complexes means there is a second or third-degree AVB. In Mobitz II, the PR intervals are the same - so no “grouped” beating, in Third degree, there is no rhyme or reason to the rhythm. In Wenckebach, the rhythm “repeats”, causing a “regularly irregular” rhythm.
A 14-month-old female is brought to your office with a 3-day history of a fever of 104 degrees rectally. Suddenly, today, the fever stopped and a rash developed. Examination reveals a diffuse, fine, maculopapular rash. Presently, the child does not appear ill. The most likely diagnosis is:
A. Rubella B. Rubeola C. erythema infectiosum (Fifth's disease) D. Roseola E. Chicken pox
Answer: D
Roseola
This is typical for Roseola. Rubella has a mild illness with a fine macular rash. Rubeola (measles) is an acute illness, the child is toxic, has conjunctivitis and Koplik spots. Fifth’s disease has mild illness with a “lacy” rash on the extremities. Chickenpox has vesicles.
At what age does the first tooth usually erupt in an infant?
A. 2-4 months
B. 6-8 months
C. 10-12 months
D. 14-16 months
Answer: B
6-8 months
The first tooth in an infant to erupt is the central incisor at the average age of 6-8 months.
A 27-year-old man comes to the clinic because of daily unilateral headaches for the past week. Episodes last 1 hour, are abrupt in onset and cessation, localized to the orbital and temporal regions, and are accompanied by lacrimation, rhinorrhea, and facial sweating. His wife says these headaches occur at night and that during the attacks her husband is restless. Which of the following is the most likely diagnosis?
A Cluster headache B Horner syndrome C Migraine D Paroxysmal hemicrania E Trigeminal neuralgia
A - Cluster Headache
A 25-year-old man comes to the emergency department because of severe pain behind his right eye. He says he has been awakened nightly by the pain over the past week. He denies nausea, vomiting, or photosensitivity. Physical examination reveals increased tearing in the right eye, but an otherwise normal neurologic examination. Which of the following is the most likely diagnosis?
A Trigeminal neuralgia B Conversion disorder C Cluster headaches D Migraine headaches E Temporal arteritis
C - Cluster Headache
A 22-year-old woman comes to the clinic because of an acute headache that feels “like a screwdiver through her temple” lasting 1 hour. She says this has happened several times in the past year, but she cannot identify a source. She rates the pain as 11/10. She additionally complains of rhinorrhea, and denies any gastrointestinal symptoms, phonophobia, dizziness, blurred vision, or photophobia. Physical examination shows ipsilateral lacrimation, ipsilateral miosis, no temporal tenderness, and no ophthalmologic pathology. Remainder of physical exam is unremarkable. Which of the following is most consistent with these symptoms?
A Tension Headache B Cluster Headache C Atypical Migraine Headache D Subarachnoid hemorrhage E Typical Migraine Headache
B - Cluster Headache
A 27-year-old man comes to the emergency department because of severe retro-orbital pain of one-day duration. He says the pain is really severe and came on somewhat suddenly. The pain is primarily concentrated behind his left eye. He also says that his eyes, especially the left, are tearing up, and his nose is dripping. He says he has had these “attacks” before. In the emergency department, he denies a history of trauma, neck stiffness, recent illness, or vision loss. No abnormalities are found on ophthalmic examination. A computed tomography scan of the head and lumbar puncture are normal. He is afebrile. Which of the following is the most likely diagnosis?
A Cluster Headache B Glaucoma C Meningitis D Migraine Headache E Subarachnoid Hemorrhage
A - Cluster Headache
__________ is a Gram-negative bacillus that may precede Guillain-Barre syndrome or reactive arthritis.
Campylobacter jejuni
A 22-year-old man comes to the emergency department because of eye pain. The patient noticed his eyes become red and painful eyes a few days ago, and symptoms seemed to be getting worse. He also complains of right knee pain. On review of systems, he is positive for increased urinary frequency. Physical examination shows bilateral conjunctivitis and bogginess of his right knee. Which of the following organisms is most likely responsible for his condition?
A Campylobacter jejuni B Chlamydia trachomatis C Escherichia coli D Neisseria gonorrhoeae E Adenovirus
B Chlamydia trachomatis
22 y/o MALE = reiter’s syndrome after MC chlamydia
A 32-year-old man comes to the clinic because of bilateral erythema, pain of the conjunctiva, and lesions on his feet. Patient says he has the sensation of “burning” and pain upon urination, and swelling of the knees. The patient rates the pain as a 7 on a 10-point scale. He is sexually active with multiple partners and does not use a barrier method of protection. Culture from a biopsy made from the patient’s foot lesions
A Campylobacter jejuni B Chlamydia trachomatis C Clostridium botulinum D Haemophilus ducreyi E Neisseria gonorrhea
patient has - KERATODERMA BLENNORRHAGICUM*** = hyperkeratotic lesions on palms / soles
Reiter’s Syndrome - reactive arthritis
MC = B - Chlamydia trachomatis
A 22-year-old man comes to the clinic because of photophobia, dysuria, and arthralgias for the past week. Patient says he had a sexually transmitted disease diagnosed a month ago, which was treated with antibiotics. Physical examination shows an erythematous eye with a watery discharge and a swollen, tender, and warm left knee on palpation. Which of the following human leukocyte antigens (HLA) types is most likely related to this condition?
A HLA B27 gene B HLA B8 gene C HLA DR2 gene D HLA DR3 gene E HLA DR5 gene
A - HLA B27 gene
A 33-year-old man comes to the office because of sore knees for the past 3 days. He says that he is also experiencing painful urination and painful, red, itchy eyes. He was treated for an infection 3 weeks ago. He has an uncle with ankylosing spondylitis. Examination shows bilateral swollen knees, with effusions. There is also bilateral injection of the conjunctivae. Which of the following microbes is the most likely cause of this patient’s condition?
A Chlamydia trachomatis B Clostridium botulinum C Neisseria gonorrhoeae D Rickettsia prowazekii E Treponema pallidum
HLA - B27 = ankylosing spondylitis, psoriasis, and reiter’s syndrome
MC = A - Chlamydia trachomatis
A 20-year-old man presents to campus health because he has had a painful, swollen right knee. His knee swelling began three weeks ago. He also complains of itchy, red eyes and a left-sided lower back ache which began two weeks ago. He denies any trauma. His medical history includes treatment for a chlamydia infection six weeks ago. Family history is significant for paternal-sided ulcerative colitis. Examination shows bilateral conjunctival injection. The patient also walks with a limp, and has a large effusion of his right knee. Which of the following is the most likely diagnosis?
A Ankylosing spondylitis B Psoriatic arthritis C Reactive arthritis D Rheumatoid arthritis E Systemic lupus erythematosus
C Reactive arthritis
HLA B27 DISEASE ASSOCIATIONS Ankylosing Spondylitis >90% = most common Reactive Arthritis 85% Reiter’s Syndrome 80% Inflammatory Bowel Disease 50% Psoriatic Arthritis 50% Whipple’s Disease 30%
Ulcerative Colitis = IBD = HLA-B27 disease
A 19 y/o male presents with acute onset of right knee pain, inflammed conjunctiva, mild burning with urination and a history of fever and diarrhea one week prior. Which of the following is the most likely diagnosis?
a. Polyarteritis Nodosa
b. Septic knee joint
c. Reactive Arthritis (Reiter’s Syndrome)
d. Systemic Lupus Erythematosus
e. Ankylosing Spondylitis
c. Reactive Arthritis (Reiter’s Syndrome)
Which one of the following disorders is associated with increased incidence of positivity for HLA-B27?
a. Pseudogout
b. Sjogrens
c. Systemic lupus erythematosus
d. Ankylosing spondylitis
d. Ankylosing spondylitis
Ossification of the annulus and fibrosis of the intervertebral disc and longitudinal ligaments (bamboo spine) appearing on x-ray are classically associated with which of the following?
a. Osteoarthritis
b. Rheumatoid arthritis
c. Degenerative disc disease
d. Reiter’s Syndrome
e. Ankylosing Spondylitis
e. Ankylosing Spondylitis `
A 27-year-old man comes to the office because of a lower back pain that radiates down his buttocks and thighs for 2 years. His medical history includes inflammatory bowel disease and type 1 diabetes mellitus. The pain is worse at night and after long periods of sitting, and gets better with movement and exercise. He also mentions that his knees have been hurting recently. Examination shows limited spinal mobility, sacroiliac joint tenderness, and reduced chest expansion. Laboratory investigations show that HLA-B27 is positive and there are raised inflammatory markers. Which of the following side effect are least likely to occur in the treatment of this condition?
A Gastritis B Leukopenia C Myositis D Renal failure E Tuberculosis
C - Myositis
Ankylosing spondylitis treatments include NSAIDs, glucocorticoids, and disease-modifying antirheumatic drugs; if this treatment fails, anti-TNF antibodies are the next treatment step. Side-effects of treatment include renal failure, leukopenia, and potentially lethal infections, including tuberculosis. In contrast, myositis symptoms are improved with NSAID and glucocorticoid therapy.
A 35-year-old woman comes to the office because of back pain for 12 years. She describes pain from her neck to tailbone. It is worse at night, sometimes even waking her from sleep. The pain subsides with movement and exercise. She also mentions having intermittent pain at the elbows, wrist, ankles and most recently, the knees. Physical exam shows restricted movements of the neck, the thoracic spine and her lumbar spine. Her ability to expand her chest was reduced, and she was tender at the sacroiliac joints. Which of the following is least likely to be found in her diagnostic work-up?
A Anti-CCP positive B Elevated C-reactive protein C Elevated erythrocyte sedimentation rate D HLA-B27 positive E Sacroiliitis
A - Anti-CCP positive
the rest can all be found on diagnostic work-up for ankylosing spondylitis
A 19-year-old man comes to the emergency room because of difficulty breathing for 6 hours. He reports that earlier that day, he was doing laundry and suddenly became short of breath. He also reports back pain for a year that is worst after waking up in the morning. He reports that his morning stiffness typically lasts for approximately an hour. Over the past few months, he has noticed pain in his hips intermittently but did not have insurance to see a doctor so he ignored it. Chest and spine X-rays are obtained. Which of the following is associated with the most likely diagnosis?
A HLA-B27 positivity B HLA-DR3 positivity C Positive ANA D Positive cyclic citrullinated peptide E Positive rheumatoid factor
A HLA-B27 positivity
A 17-year-old girl, gravida 2, para 1, comes to the obstetrics and gynecology clinic for routine prenatal care. She has a history of Hashimoto thyroiditis. She is currently 4 weeks’ pregnant. Which of the following complications is the newborn at risk for if the mother’s thyroid hormone concentrations are not properly monitored?
A Anencephaly B Caudal Regression Syndrome C Intellectual Disability D Macrosomia E Spina Bifida
C Intellectual Disability
Congenital hypothyroidism is one of the leading causes of intellectual disability in the world. All newborns in the United States are screened for hypothyroidism.
Congenital hypothyroidism, also known as cretinism, is one of the leading causes of intellectual disability in the world. During pregnancy, the mother has increased iodine requirements to ensure a euthyroid state because of the developing fetus
A 16-year-old girl comes to the clinic because of worsening fatigue for the past month. She says she has always been the smallest in her class but began puberty at age 14. Her height is now 160 cm (5 ft 3 in). While she keeps up with her class in terms of grades, she feels constantly tired despite having sufficient time for fun and rest. Physical examination shows normal skin color and texture, no periorbital edema, and no visible thyroid on extension of the neck. Her serum thyroxine, thyroid-stimulating hormone, and thyroglobulin are 3 μg/dL, 25 μU/mL, and 3ng/mL respectively. Ultrasound shows no thyroid tissue at the thyroid bed. Computed tomography shows a hyperdense soft tissue mass located at the retroesophageal superior mediastinum. Which of the following is the most likely diagnosis?
A Dyshormonogenic Goiter B Ectopic Thyroid C Iodine deficiency D Lingual thyroid E Thyroid Agenesis
TSH = 25 (very high; normal = 0.4 - 4) T4 = 3 (low; normal = 4.5 - 11)
Hypothyroidism
B - Ectopic Thyroid
Ectopic thyroid is displacement of thyroid tissue from its normal location at the lower anterior neck. The most common cause of congenital hypothyroidism is thyroid dysgenesis; the most common form of thyroid dysgenesis is ectopic thyroid; the most common form of ectopic thyroid is lingual thyroid.
A 30-year-old woman comes to the office because of fatigue. She reports dry skin, increased hair loss, and constipation. She says that she has gained more than 4.5-kg (10-lb) over the past few months, and she has missed her period for the last two months. Her respirations are 12/min, her blood pressure is 115/92 mmHg, and her BMI: 28 kg/m2. Obtaining which of the following laboratory values would be the next best step in diagnosing this patient?
A Serum TPOAb (Anti-thyroid peroxidase antibody) B TSH and free T4 C Human chorionic gonadotropin (hCG) D T3 E Free T4
B TSH and free T4
When working up a patient for hypothyroidism, the first step is to obtain TSH and free T4 laboratory values. The serum TSH would be high, and the free T4 will help determine the degree of hypothyroidism.
A 36-year-old man comes to the office complaining of feeling cold for the past 7 months. He says that he has been using less air conditioning this summer and is always dressed in layers at work. He reports a 3.6-kg (8-lb) weight gain over the past year and says he notices some blood on the toilet paper occasionally from straining to pass his stools on the toilet. Which of the following is the most appropriate initial test to order?
A Fine needle aspiration biopsy B Radioiodine scan C Thyroid stimulating hormone (TSH) level D Total thyroxine (T4) level E Total triiodothyronine (T3) level
C Thyroid stimulating hormone (TSH) level
The best screening test for suspected hypothyroidism is a TSH level. TSH would be elevated in the case of a primary hypothyroid disease such as Hashimoto thyroiditis.
The best screening test for suspected hypothyroidism is a TSH level. Levels of TSH indicate whether there is primary thyroid disease; TSH would be elevated in the case of a primary hypothyroid disease such Hashimoto thyroiditis, as in this patient. Alternatively, TSH levels would be low in other types of primary thyroid disease, such as Graves disease, in which there is an overproduction of thyroid hormone and thus symptoms of hyperthyroidism.
A 42-year-old woman presents to her primary care physician with a chief complaint of fatigue. She says that she has been sleeping more than usual. She has gained 20 pounds over the past year and has no energy to exercise. On physical exam, she appears tired and her skin is dry. Her sclerae are white. When you put her hand next to yours, you notice a difference in the hue of her skin (on the left) when compared to your own (on the right). Which of the following is most likely to be increased in this woman’s serum?
A Conjugated bilirubin
B Beta-Carotene
C Retinol
D Zinc
B Beta-Carotene
NOT A = This patient’s skin is yellow, which supports the possibility of jaundice, but her sclerae are white. Scleral icterus would suggest hyperbillirubinemia.
Carotonemia is yellowing of the skin associated with beta-carotene excess and can be seen in hypothyroidism.
Beta-carotene builds up in the blood due to lack of thyroxine (T4), which normally speeds the conversion of beta-carotene to retinol
A 71-year-old woman comes to the emergency department because of altered mental status. She was brought by a patrolman who found her wandering next to the highway. She was unable to answer questions and collapsed in transit. Her temperature is 33°C (91.4°F), pulse is 40/min, respirations are 12/min, blood pressure is 80/50, and oxygen saturation is 85% on room air. Physical examination shows decorticate posturing, incomprehensible speech, eyes opening to pain, dry hair, coarse and waxy skin, and non-pitting edema around the face and all extremities. Electrocardiogram shows sinus bradycardia. Laboratory studies show:
Calcium: 9.0 mg/dL Hematocrit (female): 34% Potassium: 4.0 mEq/L Sodium: 120 mEq/L TSH: 6.0 μU/mL Thyroxine (T4): 3.0 μg/dL Triiodothyronine (T3): 100 ng/dL
Which of the following is the most likely diagnosis?
A Myxedema Coma B Pheochromocytoma Crisis C Septic Shock D Tertiary Hyperparathyroidism E Thyrotoxic storm
normal TSH = 0.4 - 4 (6 = high!)
normal T4 = 4.5 - 11.2 (3 = low!)
normal T3 = 100 - 200
= Hypothyroidism
A Myxedema Coma
Myxedema refers to two distinct concepts:
Severe hypothyroidism that can lead to decreased temperature, blood pressure, breathing, and responsiveness
A form of edema due to deposition of connective tissue components
A 67-year-old man comes to clinic because of “forgetfulness” for the past 3 months. His daughter brought him, saying he misplaces his keys, loses his bank card, and forgets to turn off stoves. The patient also complains of constipation and weakness for the past 3 days. His temperature is 36.4°C (97.6°F), pulse is 55/min, respirations are 18/min, and blood pressure is 138/90 mm Hg. Physical examination shows dry, cold, and puffy skin with no other abnormalities. Which of the following is the most likely explanation for this patient’s condition?
A Alzheimer disease B Creutzfeldt-Jakob disease C Delirium D Hypothyroidism E Lewy Body Dementia
D Hypothyroidism
Hypothyroidism presents many symptoms akin to dementia. If caught early it can be treated and reversed. Typical signs and symptoms include low heart rate and temperature, cool extremities, cold intolerance, weight gain, fatigue, constipation, and dementia.
A 25-year-old woman presents to the emergency department with recent mood changes and increased feelings of fatigue. She also reports cold sensitivity. Her symptoms have gradually worsened over the past few months. Her vital signs are normal. Laboratory studies show a normal complete blood count and basic metabolic panel with an elevated TSH. Which of the following lab tests is most appropriate in guiding the management of this patient?
A Serum TPOAb (Anti-thyroid peroxidase antibody) B Serum free T3 (triiodothyrine) C Serum free T4 (thyroxine) D Serum total T4 E Serum total T3
C Serum free T4 (thyroxine)
TSH and free T4 levels are the two most useful laboratory studies to obtain in the evaluation of a patient with suspected thyroid disease.
A 3-month-old infant of immigrant parents is brought to the pediatrics clinic because of intermittent choking, constipation, lethargy, hoarse cry, and a suspicion that she is “floppier” than other babies since her birth. The patient is in the 15th percentile for height and 25th percentile for weight. Physical examination reveals an infant with a protuberant abdomen, dry skin, dry, brittle hair, and a low hairline. Parents report that her birth was normal. Which of the following is the most appropriate next step in management?
A Administration of the flu vaccine B Karyotype test C Heptavalent Botulinum Antitoxin D Return for her 4-month well child check E Administration of thyroxine
E Administration of thyroxine
Congenital hypothyroidism often presents with hypotonia, lethargy, macroglossia, large fontanelles, and dry skin. Early treatment with thyroid hormone is indicated.
A 35-year-old woman, gravida 3, para 2, comes to the clinic for her initial prenatal visit. She has hypothyroidism secondary to Hashimoto’s thyroiditis that is treated with levothyroxine. She is currently asymptomatic, and her TSH is 2.5 mU/L. Which of the following is the most appropriate management of her hypothyroidism as her pregnancy progresses?
A Decrease her levothyroxine dose
B Discontinue levothyroxine and begin liotrix
C Discontinue levothyroxine until after delivery
D Increase her levothyroxine dose
E Maintain levothyroxine dose
TSH normal values = 0.4 - 4.0
TSH values desired if already on Synthroid = 0.4 - 3.0
D Increase her levothyroxine dose
Although the patient is currently asymptomatic and her TSH is within normal limits (0.4-4.0 mU/L), thyroid hormone requirements increase during pregnancy to meet increased metabolic needs.
Which of the following ADHD medications has a warning that there may be an increased risk for suicide when taking this drug?
a. Guanfacine (Intuniv)
b. Lisdexamfetamine (Vyvanse)
c. Atomoxetine (Strattera)
d. Methylphenidate (Methylin)
c. Atomoxetine (Strattera)
A 25-year-old woman with a history of substance abuse is diagnosed with ADHD. Which of the following is the treatment of choice?
a. Lisdexamfetamine (Vyvanase)
b. Dextroampetamine (Dexedrine)
c. Methylphenidate (Ritalin)
d. Atomoxetine (Strattera)
d. Atomoxetine (Strattera)
if there is a hx of substance abuse in household members, can prescribe a non-stimulant therapy
What are the main neurotransmitters involved in the pathogenesis of ADHD?
a. Serotonin and GABA
b. Dopamine and serotonin
c. Dopamine and norepinephrine
d. Gluatamate and norepinephrine
c. Dopamine and norepinephrine
A 9-year-old boy is brought in for evaluation for his disruptive behavior at school. He is reported to have difficulty remaining seated, blurting out answers to questions, difficulty awaiting turns, talking excessively, intrudes on others by butting into games or activities and does not pay attention to details in his school work. What is the most likely diagnosis?
a. Generalized anxiety disorder with impulsivity
b. ADHD predominantly hyperactive/impulsive
c. ADHD predominantly inattentive
d. Oppositional defiant disorder
b. ADHD predominantly hyperactive/impulsive
Which one of the following is a contraindication to initiation of stimulant medication for the treatment of ADHD?
a. Migraine headaches
b. A family history of myocardial infarction in a maternal grandparent
c. VSD repair as an infant
d. BMI of 18
e. Irritable bowel syndrome
c. VSD repair as an infant
An 8-year-old boy who has just been diagnosed with ADHD comes in to the clinic. The family would like to administer the medication before school, avoid dosing at school (mid-day), he can swallow pills and they would like the medication to last through the school day which is 7 hours. Which of the following is the treatment of choice?
a. Guanfacine (Tenex)
b. Methylphenidate SR (Ritalin SR)
c. Methylphenidate transdermal (Daytrana)
d. Atomoxetine (Strattera)
b. Methylphenidate SR (Ritalin SR)
A 10-year-old boy is diagnosed with ADHD. His parents wish to avoid stimulant medications. Which one of the following medications would be most appropriate first line therapy for this patient?
a. Atomoxetine (Strattera)
b. Guanfancine (Tenex)
c. Clonidine (Catapres)
d. Imipramine (Tofranil)
a. Atomoxetine (Strattera)
A 9-year-old girl is brought in for follow up for ADHD. She has been on amphetamine-dextroamphetamine (Adderall) for 3 months and reports improvement in her ability to concentrate at school. She has also experienced a decreased appetite and weight loss of 6 pounds since starting this medication. Her present BMI is 19. Which of the following is the most appropriate course of action at this time?
a. Discontinue the medication
b. Supplement with a daily multivitamin
c. Have her take the medication an hour earlier in the day
d. Encourage drug holidays on the weekends and school breaks
d. Encourage drug holidays on the weekends and school breaks
Which of the following agents is effective in the treatment of Parkinson’s?
A. penicillin B. amantadine C. erythromycin D. acyclovir E. cephalexin
Answer: B - Amantadine
Amantadine (Symmetrel) is effective (somewhat). It is also used in treating Influenza A specifically. Another choice is selegiline, and levodopa/carbidopa. The other antibiotics listed here are not useful in the treatment of Parkinson’s disease.
The most common cause of preventable blindness in the United States is:
A. Macular degeneration B. Cataracts C. Retinal detachments D. Glaucoma E. Trauma
D. Glaucoma
The wording is intentionally difficult in this question. Macular degeneration, retinal detachments, and trauma are not preventable. Cataracts are “curable”, but the cataract itself is probably not “preventable.” Chronic glaucoma can be treated to prevent blindness this is a matter of semantics.
Laboratory report: elevated serum free t4; low serum TSH level. Which of the following is the MOST likely diagnosis?
A. Primary hypothyroidism B. Secondary hypothyroidism C. T3 thyrotoxicosis D. Graves disease E. Chronic Hashimoto's thyroiditis
Answer: D - Graves disease
High T4 (hyperthyroid), with a low TSH, indicates that the pituitary is working normally, and the thyroid is hyperactive i.e. Grave’s disease. Primary hypothyroidism (Hashimoto’s) is low T4 with a high TSH. Secondary hypothyroidism (from pituitary disease) is a low T4 with a low TSH.
The results of a barium esophagram reveal esophageal webbing in a woman with long-standing iron deficiency anemia. Her diagnosis is:
A. Barrett's esophagus B. achalasia C. Plummer-Vinson syndrome D. Dresslers syndrome E. Ogilvies syndrome
Answer: C - Plummer-Vinson syndrome
Plummer-Vinson is something to know for those who want a 100% on the exam. There is no need to memorize this. More importantly, webbing in the distal esophagus with no other symptoms but dysphagia would be a Schatzki’s ring (much more common). Barrett’s causes GERD symptoms and dysplasia of the esophageal mucosa. Achalasia is a primary neurologic problem of the esophagus causing dysphagia.
A 41 y/o female presents to PCP complaining of difficulty swallowing her food x last 7 months. Has lost 15 pounds unintentionally and has been tired. Drinks 1 glass of wine/week and 5 pack year history of smoking but quit 20 years ago. Denies taking any medication. Upper endoscopy is ordered. Labs are as follows:
WBC count = 6050 /mm3 RBC count = 2.5 million Hemoglobin = 10.2 g/dL MCV = 72 MCHC = 28% hb/cell RDW = 16.8% Platelet count = 490,000
Which sign/symptom is most likely in this patient?
a) atrophic glossitis
b) esophageal adenocarcinoma
c) nocturnal cough and dyspnea
d) painful esophageal mucosal lacerations
e) painless esophageal bleeding
1) difficulty swallowing food (dysphagia)
2) esophageal webbing on picture
3) MCV = low = microcytic anemia = iron. RDW = high (11.5 - 14.5) = iron deficiency anemia
= a) atrophic glossitis
Patient has plummer vinson’s syndrome! = dysphagia, esophageal webbing, glossitis, iron deficiency anemia
A 42-year-old woman who recently emigrated from Norway comes to the free clinic because of progressive trouble swallowing for the past few months. She says that when she tries to swallow bread, it often feels like it gets stuck before it reaches her stomach. She also complaints of fatigue and shortness of breath after walking up the stairs to her 2nd-floor apartment that requires her to rest for several minutes to recover. Her current medications include an oral contraceptive pill. Her temperature is 37.0°C (98.6°F), pulse is 88/min, respirations are 17/min, and blood pressure is 110/66 mm Hg. Physical examination shows a thin, pale woman with a smooth, red, and shiny tongue. Laboratory studies show a hemoglobin of 9.1g/dL and a leukocyte count of 9,000mm3. Barium esophagram shows multiple esophageal webs, as shown below. Which of the following is the most likely diagnosis?
A Achalasia B Esophageal adenocarcinoma C Gastroesophageal reflux disease D Plummer-Vinson syndrome E CREST syndrome
D Plummer-Vinson syndrome
Plummer-Vinson syndrome is a condition characterized by the triad of dysphagia, iron-deficiency anemia, and esophageal webs that is often associated with glossitis as well. Patients are at an increased risk of squamous cell carcinoma of the esophagus, and initial treatment is iron-supplementation and relief of dysphagia.
normal hemoglobin = 12-18 (9.1 = LOW)
normal leukocyte = 4,000 - 10,000 (9,000)
iron deficiency anemia
glossitis
dysphagia
Which of the following disorders is described as a milder pattern of mood swings that does not reach the severity of bipolar disorder but does include brief depressive and manic episodes?
a. Anhedonia
b. Anxiety disorder
c. Dysthymia
d. Cyclothymia
d. Cyclothymia
A 16-year-old girl comes to the pediatrician for a routine check-up. She denies feeling depressed but admits fighting with her mother more often. She reports that she has not menstruated in four months. Physical examination is normal except for excessive growth of lanugo body hair. Which of the following will be normal in this patient?
A Height B Complete blood count C ECG D Lipid panel E X-ray
A - Height
Not CBC - Complete blood count may show signs of anemia and leukopenia. - low platelets + RBCs
Not EKG - Bradycardia, hypokalemia changes; electrolyte imbalances may lead to cardiac abnormalities
not Lipid Panel - could lead to impaired liver function. This is could cause hypercholesterolemia and elevated triglycerides.
not XRAY - osteoporosis
A 17-year-old female comes to the emergency room because of a suspected colles fracture after tripping and falling in the parking lot at school. Her pulse is 64/min and blood pressure is 104/72 mm Hg. Her height is 162 cm (5 ft 4 in) and weighs 45.4-kg (100-lbs). She confirms her last menstrual period was more than 3 months. Urine pregnancy test shows no sign of pregnancy. Which of the following is her diagnosis?
A Premature ovarian insufficiency B Ewing Sarcoma C Bulimia nervosa D Osteogenesis imperfecta E Anorexia nervosa
E Anorexia nervosa
bradycardia
hypotension
fracture - osteoporosis
amenorrhea
Amenorrhea results from disorders in the hypothalamic-pituitary-ovarian axis in which levels of follicle-stimulating hormone (FSH) and luteinizing hormone (LH) are low despite low levels of estrogen. Due to this, reversion to the prepubertal state occurs, and the LH response to gonadotropin-releasing hormone (GnRH) is blunted. This blunted response is insufficient to maintain menstrual integrity, and amenorrhea results
A 14-year-old girl comes to the hospital because of anorexia nervosa. She acknowledged that she has not eaten anything for the past 4 days. Physical examination is notable for autonomic dysreflexia with positive chvostek and trousseau signs. She also complains of being too fat. Which of the following is associated with her condition?
A Low levels of chloride B High levels of chloride C Low levels of magnesium and calcium D High levels of sodium E Low potassium levels
C Low levels of magnesium and calcium
Chvostek and Trousseau signs = hypoparathyroidism –> low levels of calcium + magnesium
Anorexia nervosa (AN) is characterized by hypotension, bradycardia, hypothermia, dry skin, hypercarotenemia, lanugo body hair, acrocyanosis, atrophy of the breasts, swelling of the parotid and submandibular glands, peripheral edema, and thinning of the hair.
An increase in blood urea nitrogen (BUN) levels, which reflects a level of dehydration and a decreased glomerular filtration rate (GFR), is present. Electrolyte imbalances are secondary to vomiting, and potassium is most often affected. In cases where hypomagnesemia and hypocalcemia are severe, chvostek and trousseau signs may be positive
Low levels of sodium are associated with anorexia nervosa. This causes low blood pressure and affects renal function. Edema can be observed.
Low potassium levels are seen in anorexia nervosa, but would cause prolonged QT, flattened T waves, and arrhythmias
In anorexia nervosa, chloride levels are low. This can result to hyperirritability, tetany or muscular excitability, slowed respirations, and hypotension secondary to fluid loss.
A 19-year-old woman comes to the clinic because of amenorrhea. She acknowledged being on a dieting spree with episodes of induced vomiting. Her BMI is 15 kg/m2. Pregnancy test shows negative. Endocrine and coagulation studies are normal. Genital cultures and pelvic ultrasound are non-contributory. Which of the following is her likely diagnosis?
A Anorexia nervosa B Hyperthyroidism C Hypothyroidism D Pituitary adenoma E Primary amenorrhea
A) Anorexia nervosa
amenorrhea
BMI of 15 (normal 18.5 - 24.5)
negative pregnancy test
dieting spree with induced vomiting = binge/purge type of anorexia
An 18-year-old Caucasian girl comes to the hospital because of poor feeding habits. She explains she tries to avoid extra calories. On review of systems, she denies headache but admits to feeling fatigued and not having a menstrual period for the last 3 months. When asked to remove her baggy clothes for the physical exam, lanugo hair is observed. Which of the following is likely to be found on serum studies?
A Hypercalcemia B Hyperkalemia C Hypercarotenemia D Hypernatremia E Hypermagnesemia
C Hypercarotenemia
Anorexia nervosa (AN) is usually characterized by low levels of body electrolyte and protein. However, due to impaired liver functions, there could be hypercholesterolemia and hypercarotenemia.
An 18-year-old girl comes to the student health office because of irritableness and concerns about decision-making. On physical examination she appears very thin, with decreased muscle mass and dry skin. She is hypotensive with resting bradycardia. She denies any episodes of induced vomiting. Which of the following is associated with her condition?
A Calluses on hands or fingers B Dental erosions C Hirsutism D Primary amenorrhea E Secondary amenorrhea
E Secondary amenorrhea
Primary amenorrhea is the failure of menses to occur by age 16 years, in the presence of normal growth and secondary sexual characteristics. If by age 13 menses has not occurred and the onset of puberty, such as breast development, is absent, a workup for primary amenorrhea should start
Secondary amenorrhea = lack of period DUE TO anorexia
An adolescent female comes to the dentist because of acidic erosion of her teeth (photograph is shown below). She was observed to be emaciated with dry skin, hypercarotenemia, and lanugo body hair. She was also hypotensive with dry mucous membranes. An ECG showed prolonged QT interval. Which of the following electrolyte and acid/base disturbance is associated with her illness?
A Hyperkalemic metabolic acidosis
B Hyperkalemic metabolic acidosis and acute compensatory respiratory alkalosis
C Hypokalemic hypochloremic metabolic alkalosis
D Low serum calcium leading to arrhythmia
E No electrolyte or acid/base disturbance
C Hypokalemic hypochloremic metabolic alkalosis
An 18-year-old woman comes to the hospital because of poor feeding habits. She explains she tries to avoid extra calories. On review of systems, she denies headache but admits to feeling fatigued and not having a menstrual period for the last 3 months. When asked to remove her baggy clothes for the physical exam, lanugo hair is observed. Which of the following is ideal in managing her condition?
A Administering olanzapine B Cognitive-Behavioral Therapy (CBT) C Prescribing bupropion D Prescribing orlistat E Prescribing sertraline
avoids eating
fatigue
amenorrhea
lanugo
= anorexia nervosa
ANSWER = CBT (COGNITIVE BEHAVIORAL THERAPY)
Anorexia nervosa (AN) is an eating disorder which is mainly treated by family therapy and psychotherapy management. If this fails, olanzapine could be prescribed.
olanzapine (Zyprexa) = antipsychotic - weight gain
buproprion (Wellbutrin) = antidepressant
orlistat = weight loss drug (X)
sertraline (Zoloft) = antidepressant
A 26-year-old woman comes to the clinic because of amenorrhea. On examination, the woman has a BMI of 16. There is no abdominal tenderness or guarding. Vaginal and bimanual exams are grossly normal. A pregnancy test is negative. Serum levels of FSH, LH, TSH, prolactin are low. A basic metabolic panel returns results consistent with hypokalemia, hypochloremia, and compensated anion gap metabolic alkalosis. Which of the following is her likely diagnosis?
A Renal disease B Cervical stenosis C Pituitary adenoma D Anorexia nervosa E Androgen tumor
D Anorexia nervosa
BMI 16 = moderate
amenorrhea
hypokalemia, hypochloremia, metabolic alkalosis
Anorexia nervosa (AN) may present with low levels of FSH, LH, TSH, and prolactin. However, if bulimic episodes are also involved, these hormones may be normal. The patient’s history usually gives more direction.
A 15-year-old female comes to the clinic for her regular check-up. She is 165 cm (5 ft 5 in) tall and weighs 36.8 kg (85 lbs). She acknowledges taking low low-calorie diet with possibility of skipping meals. She has not had a period in 3 months, and she is worried that she might be pregnant. Serum beta-human chorionic gonadotropin is negative. Which of following is most appropriate in management?
A Discuss a nutrition plan and counseling for her anorexia nervosa
B Discuss a nutrition plan and counseling for her bulimia nervosa
C Tell her that the pregnancy test is probably false negative because she has not menstruated
D Performing endocrine work-up for secondary amenorrhea.
E Prescribing oral contraceptives
A Discuss a nutrition plan and counseling for her anorexia nervosa
restrictive anorexia nervosa
amenorrhea
A 16-year-old boy comes to the hospital because of extreme dieting and heavy exercise. The boy complains of being scared of gaining weight despite being told that his BMI of 16 is dangerous for his health. He has wasted features and fine downy hair. His family members are really concerned about him. What is the next best step in management?
A Olanzapine B Family therapy C Sertraline D Haloperidol E Amiodarone
B Family therapy
restrictive anorexia nervosa
The first line approach is family therapy, followed by cognitive behavioral therapy.
A 6-year-old girl comes to the office because of behavioral changes that concern her mother. The mother says that her daughter has repeatedly been sent to the principal’s office because she disrupts the class by excessive talking, interrupting other children, not raising her hand and not staying in her seat. She does not follow instructions. At home, she can not sit still and has a difficult time completing her usual chores, often disregarding anything her parents tell her. She estimates this behavior started about a year ago and thought it would just be a phase. Which of the following is most commonly associated with the most likely diagnosis?
A Autism B Chromosomal abnormalities C Low IQ D Oppositional defiant disorder E Rett syndrome
The most likely diagnosis is attention deficit hyperactivity disorder (ADHD). These children have trouble paying attention, are easily distracted and have difficulty following instructions. These children also must demonstrate these symptoms in at least two settings.
Oppositional defiant disorder (ODD) is a common co-morbidity. Children with ODD display hostile behavior towards authority figures. The other choices are not commonly associated with ADHD.
A 7-year-old boy comes to the office because has been failing to do schoolwork and his mother is concerned about his ability to focus. He has difficulty paying attention in school, staying quietly seated during class, and often interrupts without waiting for his turn to speak. Which of the following criteria must be present in order to diagnose attention deficit hyperactivity disorder?
A Evidence of social skill impairment
B Overwhelming fear and worry in response to schoolwork
C Signs of impulsivity at school only
D Sudden rapid motor tics that persist for over a year
E Symptoms present for at least 6 months
E Symptoms present for at least 6 months
A 28-year-old woman comes to the office because she wants to be able to concentrate better at work. She says that she cannot get work done because she is easily distracted. She recently got into legal trouble for fighting with a stranger in a movie theater. She reports trouble concentrating in the past which resulted in poor grades in school, and has been taking Ritalin for this condition. Which of the following is a characteristic of her diagnosis?
A Below-average intelligence B Decreased size of frontal lobes C Onset before age 5 D Poor social interactions E Symptoms always seen in adulthood
Attention deficit hyperactivity disorder (ADHD) is a developmental disorder characterized by limited attention span and poor impulse control. It is associated with a decrease in frontal lobe volume or metabolism, but does not result in a lower intelligence level.
A 40-year-old woman was just recently diagnosed with attention deficit hyperactivity disorder (ADHD). She says that she can become very focused when at work, but when not at work her thoughts are very scattered. When asked if she has experienced this in the past, she reports trouble concentrating the same way ever since she was a young student in elementary school. She has recently starting taking methylphenidate for her symptoms. In order to provide patient education on her new treatment, which of the following is true about this treatment?
A Hypotension is a common side effect
B Chronic use can lead to tardive dyskinesia
C Appetite suppression is a common side effect
D Bupropion is more effective in adults with ADHD than methylphenidate
E Sedation is a common side effect
C Appetite suppression is a common side effect
NOT A) Methlyphenidate is a CNS stimulant, and thus works to increase the levels of catecholamines in the synaptic cleft. Hypertension and tachycardia are common side effects of this stimulant medication.
NOT B) Tardive dyskinesia is characterized by uncontrolled stereotypical movements in the lower face, and can be seen as a result of long-term antipsychotic use.
NOT D) Stimulant medications and non-stimulant medications, like anti-depressants, are generally considered equally effective. However, stimulants and atomoxetine are still considered first-line treatment, which remains the same in children and adults
NOT E) CNS stimulants like methylphenidate and dextroamphetamine work by increasing the levels of catecholamines in the synaptic effect. Thus, one of its clinical uses is treating narcolepsy.
A mother brings in her 7-year-old son because she is worried about his behavior after his teacher called home. The boy has not been doing his assignments and frequently tells his teachers that he forgets about them. In addition, he tends to have a difficult time sitting still in class and will disrupt the other children. The behavior has been going on for about 8 months, but it is now getting worse so this was brought to the mother’s attention. His mother was surprised to hear about these issues since she has not noticed bad behavior at home. However, she does state that she needs to repeat herself several times when asking her son to complete his chores. What is the most likely diagnosis?
A Schizoid personality disorder B Attention deficit hyperactivity disorder C Conduct disorder D Oppositional defiant disorder E Antisocial personality disorder
B Attention deficit hyperactivity disorder
A 9-year-old boy comes to the office due to concern over his repeated difficulties in school. His mother explains that over the past 6 months, she has repeatedly met with his teacher due to his conduct at school. His teacher said that he constantly speaks loudly without raising his hand, does not clean up his assigned area, and does not finish his homework on time. During the office visit, the patient is constantly grabbing objects and is very irritable. The patient was prescribed methylphenidate. What is the mechanism of action of this drug?
A Monoamine oxidase inhibitor B 5-HT2 receptor antagonist C D2 receptor antagonist D Cholinesterase inhibitor E Blocks reuptake of dopamine and norepinephrine
Methylphenidate = Focalin = Stimulant =
E Blocks reuptake of dopamine and norepinephrine
A 6-year-old boy comes to the psychiatric department because of disruptive behavior that has lasted 8 months. His mother is concerned about his difficulties with schoolwork. She notes that he has always been a very active child and is easily distracted. He has been sent to the principal’s office multiple times for disrupting class and impulsive behavior towards his peers. Although the first-line pharmacotherapy for this condition is a stimulant, the mother is hesitant about using it. Which non-stimulant medication can also be used to treat this condition?
A Methylphenidate B Lorazepam C Fluoxetine D Atomoxetine E Dextroamphetamine/Amphetamine
D Atomoxetine
A 10-year-old girl comes to the office because she has had trouble concentrating in school this year. Her mother reports that she has not been doing her assignments. She tends to have a difficult time sitting still in class, following instructions, and disrupts the other children. She is not disruptive at home, but she will forget to do chores often according to her mother. Which of the following treatments should be recommended for this child?
A) Atomoxetine B) Clonidine C) Fluoxetine D) Methylphenidate E) Parental Counseling
D) Methylphenidate
Stimulant drugs are considered the first-line treatment for ADHD in children
Methylphenidate (or Ritalin) is usually the first line stimulant drugs used as medical therapy
Which of the following is most likely to be found on the laboratory evaluation of a patient admitted for treatment of anorexia nervosa?
a. Normal BUN
b. Ketonuria
c. Hypernatremia
d. Hyperkalemia
e. Decreased liver function tests (LFT)
b. Ketonuria
Which of the following is most likely to be a long-term complication associated with anorexia nervosa, even if the patient is able to regain ideal body weight after treatment?
a. Congestive heart failure
b. Pericardial effusion
c. Constipation
d. Superior artery mesenteric syndrome
e. Osteopenia
e. Osteopenia
When admitting a patient for further workup and treatment of anorexia nervosa, what laboratory finding would you most likely see?
a. Metabolic acidosis
b. Leukopenia
c. Hyperkalemia
d. Increased levels of FSH
e. Decreased serum bicarbonate level
b. Leukopenia
A 16-year-old girl comes to the office because of decreased mood and tiredness. Her mother says that she has not been doing any of her schoolwork, and often displays a sudden loss of interest in her extracurricular activities. Her pulse is 90/min and her blood pressure is 110/75. Her BMI is 22. Physical examination shows decaying enamel on teeth and drying skin. Thickened skin is found around the knuckles of her right hand. A complete metabolic panel would most likely show which of the following electrolyte disturbances?
A Hypercalcemia B Hyperkalemia C Hypocalcemia D Hypokalemia E Hyponatremia
D Hypokalemia
would see hyponatremia and hypocalcemia, but patient would have different symptoms
Although dry skin can be a sign of hypocalcemia, it is often accompanied by neurological symptoms such as headache and muscle cramping. It can be associated with many causes, including hypoparathyroidism, vitamin D deficiency, and acute renal failure.
Hyponatremia is most common electrolyte abnormality in hospitalized patients. It indicates excessive extracellular fluid relative to sodium in the body and can lead to nausea, vomiting, lethargy, and seizures.
A 19-year-old woman comes into the office because of abdominal pain and lack of energy. She says she has not been participating in activities at her college, and often feels overweight. However, her current BMI is 21. Six months ago, her BMI was 23. Physical examination shows tachycardia, erosion of dental enamel and callus formation over her knuckles. Which of the following acid-base disorders would most likely be seen in this patient?
A Metabolic acidosis B Metabolic alkalosis C Mixed respiratory and metabolic acidosis D Respiratory acidosis E Respiratory alkalosis
B Metabolic alkalosis
A 20-year-old woman comes to the emergency department because of suspected aspiration following an episode of choking. She reports abdominal pain. Physical examination shows erosions of the teeth, petechiae, and calluses on the knuckles of her right hand. At a BMI of 22.6, she weighs 140 lbs (63.5 kg) with a height of 5’6”. Her pulse is 90/min and blood pressure is 104/76. Based on the clinical findings, which of the following is the most likely diagnosis?
A Anorexia nervosa B Bulimia nervosa C Gastroesophageal reflux disease D Major depressive disorder E Premature ovarian insufficiency
B Bulimia nervosa
A 15-year-old girl comes to the office because of lethargy, irregular menses, and decreased mood. She has recently dropped out of all extracurricular activities at school and says she feels unattractive and overweight. Physical examination shows ragged-appearing enamel over the front teeth and thickened skin over her knuckles. She has a height and weight in the 95th and 85th percentile, respectively. Based on these findings, which is the most likely diagnosis?
A Anorexia nervosa B Body dysmorphic disorder C Bulimia nervosa D Cushing syndrome E Hypothyroidism
C Bulimia nervosa
A 17-year-old girl presents at her pediatrician’s office accompanied by her mother for a normal check-up. She has recently been forcibly vomiting after meals. She states that she has recently been very anxious in school and that vomiting is the only thing in life that she can control. Her vitals show no abnormalities and BMI is 22.1 kg/m2. In addition to family and cognitive behavioral therapy, which of the following is the most appropriate pharmaceutical interventions?
A Bupropion B Clonazepam C Fluoxetine D Olanzapine E Ondansetron
C Fluoxetine
A 25-year-old woman with no significant past medical history comes to the office for a routine physical. She is married with two children and complains of feeling physically and emotionally drained by her demanding career in politics, as well as her dwindling relationship with her famous husband. Although she is concerned about her weight, her current BMI is 20. Her pulse is 98/min, temperature is 99.7 o F (37.6 oC), blood pressure is 115/76, and respirations is 12/min. Physical examination reveals bilateral parotid enlargement, tooth enamel erosion, and scarring on the knuckles. A complete metabolic panel will most likely find which of the following electrolyte imbalances?
A Decreased bicarbonate B Hypercalcemia C Hyperchloremia D Hypokalemia E Hyponatremia
D Hypokalemia
A 23-year-old man comes to the university health center because of his partner’s concern about his vomiting habits. The patient says that he feels guilty after eating and often feels overweight. Physical examination shows a BMI of 22 kg/m2, pulse is 90/min, and blood pressure is 110/60 mm Hg. His teeth enamel show signs of deterioration and knuckles on his hands have calluses. Which of the following is the appropriate next step in management?
A Behavioral weight loss therapy B Couple's therapy C Fluoxetine D Haloperidol E Pregabalin
C Fluoxetine
Which of the following agents has a mechanism of action that does not involve beta-receptors?
A. epinephrine B. albuterol C. propranolol D. prazosin E. ritodrine
D - Prazosin
prazosin is an alpha-blocker. Ritodrine and epinephrine cause beta sympathetic stimulation. Albuterol is a beta agonist. Propranolol is a beta blocker
beta receptors = heart + lungs
beta 1 = heart
beta 2 = lungs
A 19-year-old woman with no significant past medical history comes in for a pre-college physical examination required by her school. She has no complaints, and her review of systems is negative. Her temperature is 37.0°C (98.6°F), blood pressure 120/80 mm Hg, pulse 80/min, and respirations 10/min. Physical examination reveals a single nodule on the left lobe of the thyroid gland. Her thyroid-stimulating hormone is within normal limits.
Which of the following is the most appropriate next step in management?
CT scan of the thyroid gland
Fine-needle biopsy
Repeat examination in 6 months
Thyroid uptake scanning
Fine-needle biopsy
The correct answer is B. The first test for a thyroid nodule is TSH; if normal, the most appropriate next step is FNA biopsy - which can be done safely and without complication. An ultrasound can be used to differentiate between solid and cystic structure and can be used to guide the needle aspiration. The FNA biopsy provides a definitive diagnosis in most cases and is the most helpful test in guiding management.
CT scan (choice A) is important in the workup of a patient who has a goiter and is complaining of obstructive symptoms such as shortness of breath or difficulty swallowing. It has no role in the initial workup of a thyroid nodule. If the patient has thyroid cancer, the CT scan of the thyroid can help stage the cancer.
Serial physical examinations (choice C) are not acceptable management. Thyroid cancer is curable in its early stages, so efforts should be made to rule out carcinoma in these patients
PEARL: A patient who has a thyroid nodule should have a full thyroid function evaluation performed. The first test performed should be the TSH; a suppressed level suggests hyperthyroidism and a hot nodule, which lessens the possibility for cancer. The next test should be a fine-needle biopsy (if the TSH is not suppressed). Histology will identify whether the patient has cancer and also will identify the type of cancer that the patient has. A thyroid scan can help to identify the nodule and whether it is hyper- or hypofunctioning.
A 58-year-old woman is found on a routine medical check-up to have a serum calcium level of 11.8 mg/dL, where the upper limit of normal is 9.5 mg/dL. Repeated determinations confirm values between 10.9 and 12.2 mg/dL, and she is found to have elevated concentrations of parathyroid hormone. She is asymptomatic, has no pertinent family history, and has no evidence of renal stones or bone disease. She is offered the option of elective parathyroidectomy, but she declines and elects to have close medical follow-up.
In addition to monitoring serum calcium levels, which of the following therapeutics should be given?
d
A 6-year-old male is brought into your office for evaluation of his gait. You note that when ambulatory, the medial aspects of his knees are far apart and his feet are unusually close together. Which of the following best describes this condition?
A. cubitus valgus B. cubitus varus C. genu valgus D. genu varum E. Legg-Calve-Perthes disease
Answer: D = Genu varum
This is genu varum (bow legs). Valgus deformity forms an “L” in the knock-kneed child and has an “L” in the middle of it (try it, it works)
A female presents complaining of a grayish, watery vaginal discharge. The KOH prep does not reveal any hyphae. Clue cells are identified on the wet mount. A paucity of WBCs are noted. What diagnosis is most likely?
A. Gonorrhea B. Chlamydia C. Candidiasis D. bacterial vaginosis E. Trichomoniasis
D. bacterial vaginosis
Clue cells mean BV
FISHY, grey, scant, THIN, STICKY
PH > 4.5 (BACTERIA = BASIC)
Wet-prep shows epithelial cells with bacteria coating on surface = “clue cells”
KOH testing causes the release of amine odor (+ whiff test)
First Line Treatment:
Metronidazole: 500 mg PO b.i.d. × 7 days*****
Metronidazole gel 0.75%: 5 g intravaginally daily × 5 days
Clindamycin 2% cream: 5 g intravaginally qhs × 7 days
You note polygonal, purple, popular lesions involving the flexor surfaces of a patient’s wrists. The patient has no other lesions. The lesions appear to follow a linear pattern of distribution. What is this phenomenon?
A. Auspitz sign B. Koebner’s phenomenon C. Darier’s sign D. Gorlin’s sign E. Gottron’s papules
B. Koebner’s phenomenon
Koebner’s phenomenon is described - it suggests psoriasis = refers to skin lesions appearing on lines of trauma / irritation
Auspitz sign is also present in psoriasis where the scales bleed when removed.
Darier’s sign is in allergic patients who develop urticaria when the skin is rubbed.
Gottron’s papules are purple papules on the fingers seen in dermatomyositis.
Gorlin’s sign is seen in Ehlers-Danlos when a patient can touch the tip of the nose with his tongue
Which of the following is most likely on history of a patient with schizoid personality disorder?
a. Lives alone and is single
b. Employed in a highly competitive sales job
c. Incarceration
d. Paranoid ideations
a. Lives alone and is single
avoid social interaction because they aren’t interested in getting to know others. Results in isolation; they want to be isolated. Find less pleasure in physical contact - like sex or holding hands. Less likely to seek out relationships than the average person
- may also have a flat affect or emotional blunting- don’t show positive or negative emotions
After watching a particular movie 5 months ago, a young man becomes infatuated with an actress in the film and believes the movie is really about their destiny to be together. He begins to stalk her and sees a conspiracy unfolding that is keeping them apart. When she continually ignores his phone calls and poems he believes he will win her love and catch her attention by assassinating the current president. He believes there is no chance that his plot will fail. He suffers from no visual or auditory hallucinations. Socially, he has few relationships and is awkward and paranoid, which causes him social anxiety. These fanciful beliefs influence his behaviors and he begins to obsess about his plan. Based on this information, which of the following is the most likely diagnosis?
A Schizoid personality disorder B Schizophrenia C Schizotypal personality disorder D Paranoid personality disorder E Schizophreniform disorder
C Schizotypal personality disorder
Schizotypal personality disorder is a cluster A personality disorder characterized by odd behavior and magical thinking. They lack delusions or hallucinations, unlike schizophrenia.
People with schizotypal PD often exhibit ideas of reference, are socially withdrawn/awkward/anxious or paranoid, and may have abnormal perceptual experiences and constricted affect. They may also dress in an odd manner. Many of these characteristics manifest in this patient.
- Ideas of Reference - belief that everything, including innocuous events or coincidences in the world, directly relate to ones own DESTINY
A 42-year-old man comes to the office for blood work because he thinks his fiancé has been “poisoning” him for the last 6 months. He thinks she is planning on killing him in order to inherit his money after they get married. When asked how he knows this, he says he hears his family members whispering about it and they are unaware he can hear them. He says that he has to constantly check to see if his fiancé’s spies are watching him. He also says that people at his work have been commenting secretively about how thin and gaunt he looks. He does not eat or sleep much because he is so busy watching his back. Which of the following is the most likely diagnosis?
A Antisocial personality disorder B Borderline personality disorder C Paranoid personality disorder D Schizoid personality disorder E Schizotypal personality disorder
C Paranoid personality disorder
Patients with paranoid personality disorder hold extremely improbable and odd beliefs that affect their functioning, but are able to function well in areas that are outside their paranoid suspicions.
A 35-year-old man comes to the office for an annual visit. He says he has been under a lot of stress lately at home and at work and is afraid that he will lose his job since his co-workers are trying to sabotage him. He does not get along with his co-workers and believes it is because he does a better job. At home, he suspects that his wife is cheating on him because he does not make enough money to support her. He denies any auditory and visual hallucinations, special powers, ideas of reference, or belief that others can hear his thoughts. His wife says he has always had a difficult time confiding in others and would hold grudges against friends and co-workers. Which of the following is the most likely explanation for this man’s symptoms?
A Narcissistic personality disorder B Paranoid personality disorder C Paranoid schizophrenia D Schizoid personality disorder E Schizotypal personality disorder
B Paranoid personality disorder
Patients with paranoid personality disorder are suspicious of others, display pervasive distrust, and often interpret motives as malicious. They tend to read hidden threatening meanings into remarks and react angrily to perceived insults. The paranoid thoughts of these patients are merely suspected and not believed with certainty.
A 45-year-old man comes to the physician because of concerns about his prostate after experiencing frequent urination during the day and night. He very rarely sees his primary care physician but has been a patient there for several years. He is a quiet man and works as a laboratory technician usually during the overnight shift. He is single and has never had a relationship before. He has no close friends, but he expresses no desire to have friends. There is no evidence of hallucinations or a thought disorder. Which of the following is statements most accurately explains the patient?
A He most likely has a Cluster A personality disorder
B He most likely has a Cluster B personality disorder
C He most likely has a concomitant substance abuse disorder
D He most likely has a genetic association with anxiety disorders
E He most likely has a pattern of disregard for the rights of other people
A He most likely has a Cluster A personality disorder
= Schizoid personality disorder
Patients with a schizoid personality disorder will be detached from others and have little desire for close relationships. People with this disorder prefer to be alone and take pleasure in few activities.
A 35-year-old man comes to the office for an annual visit. He says he has been under a lot of stress lately at home and at work. He does not get along with his co-workers and believes it is because he does a better job. He suspects that his wife is cheating on him with the neighbor because he does not make enough money to support her. He denies any auditory and visual hallucinations, special powers, ideas of reference, or belief that others can hear his thoughts. His wife says that he has always had a difficult time confiding in others and would hold grudges against friends and co-workers. Which defense mechanism is this man most likely to display?
A Acting out B Denial C Dissociation D Projection E Regression
D Projection
Paranoid personality disorder is a cluster A personality disorder. Projection is the major defense mechanism seen in this disorder and is characterized by attributing thoughts or feelings to others.
Projection is the major defense mechanism seen in this personality disorder, in which patients take an unacceptable internal impulse, wish, or desire and attributes it to an external source. For example, a man may think about cheating on his wife so he blames his wife for having an affair. Another example would be a boy who talks about his stuffed animal as having certain feelings, which are really what the boy feels. Projection is an immature defense mechanism and paranoia tends to result from the use of projection
A 30-year-old woman comes to the clinic for an annual checkup. She is dressed in mismatching clothing and believes that she has telepathic abilities. Her family has previously stated that she has been superstitious since she was a child, and she lacked close friendships due to her peculiar behavior. Which of the following is this patient’s most likely personality disorder?
A Avoidant personality disorder B Borderline personality disorder C Histrionic personality disorder D Schizoid personality disorder E Schizotypal personality disorder
E Schizotypal personality disorder
Schizotypal personality disorder is a cluster A disorder characterized by eccentric beliefs, magical thinking, and constricted affect. These patients often lack close relationships.
Schizotypal personality disorder is characterized by at least five of the following:
1) odd beliefs or magical thinking
2) perceptual experiences
3) odd thinking and speech
4) suspiciousness of others
5) constricted affect
6) odd behavior
7) lack of close friends
8) social anxiety.
These patients often have BIZARRE ATTIRE, SPEECH AND BEHAVIOR. They may wear MISMATCHING CLOTHING and say UNUSUAL / MADE-UP WORDS. They often have IDEAS OF REFERENCE in which they interpret events that are completely unrelated as having specific meaning toward themselves. The treatment of choice, as with all personality disorders, is psychotherapy.
- Ideas of Reference - belief that everything, including innocuous events or coincidences in the world, directly relate to ones own destiny
You’re examining a 35-year-old man who is exhibiting acute psychotic behavior. He also has delusions and hallucinations. He has a history of a hypertrophic cardiomyopathy as well. Which of the following medications would you be very hesitant to give this man?
(A) Haloperidol (Haldol) (B) Olanzapine (Zyprexa) (C) Lithium carbonate (Lithobid) (D) Oxazepam (Serax) (E) Diazepam (Valium)
(A) Haloperidol (Haldol)
Haloperidol (Haldol) is a typical antipsychotic that can cause cardiac arrhythmias, specifically by prolonging the QT interval. You wouldn’t give it to someone with a hypertrophic cardiomyopathy. A good choice to use for this patient would be olanzapine (Zyprexa).
- Lithium carbonate is used for bipolar disorder
- Oxazepam and diazepam are used for anxiety and panic disorders. These other meds may be used adjunctively; although the person still needs to be given a medication to control his or her psychotic behavior).
You’re seeing a 35-year-old woman for evaluation of major depressive disorder. You want to begin treatment with an antidepressant. Which of the following conditions do you need to exclude before you treat her?
(A) Panic disorder (B) Seasonal affective disorder (C) Borderline personality disorder (D) Bipolar disorder (E) Dementia
(D) Bipolar disorder
In anyone you think has major depressive disorder, make sure you consider that the person may have bipolar disorder. By giving a bipolar person an antidepressant, you can unmask a manic episode
Choice (A), panic disorder, wouldn’t be a concern here, and Choice (B), seasonal affective disorder, makes no sense. Nothing in the question suggests that her depression is related to seasons of the year (for example, better in June than in January). Note that Choice (C), borderline personality disorder, can be associated with depression. The woman in the question is typically too young to have dementia, Choice (E), and nothing in the question makes you think that she does.
You’re seeing a 25-year-old man in the ER who was recently started on metoclopramide (Reglan). He presents with a very high fever and a change in mental status. On physical examination, his muscles are rigid. His body temperature is 40.0°C (104°F). You obtain a stat CPK level, and it is 50,000. This man is in severe trouble. What is the most likely diagnosis?
(A) Serotonin syndrome (B) Tyramine reaction (C) Neuroleptic malignant syndrome (NMS) (D) Drug overdose (E) Bacterial infection
(C) Neuroleptic malignant syndrome (NMS)
This man has neuroleptic malignant syndrome (NMS), which you usually see after someone takes a neuroleptic or antipsychotic. This person took metoclopramide (Reglan). He presents with fever and a change in mental status. His muscles are rigid because of the medication. A person with neuroleptic malignant syndrome has an elevated CPK level.
Choice (A) is wrong because serotonin syndrome (SS) occurs with medications that can cause excessive serotinergic activity. Examples of such medications include SSRIs, tricyclics, certain over-the-counter medications, and anti-Parkinson’s medications. Symptoms can include confusion, fever, and tachycardia. You usually see Parkinson’s and hyperactive reflexes with SS, as compared to the muscle rigidity you seen in neuroleptic malignant syndrome.
A tyramine reaction, Choice (B), concerns the interaction of a MAO inhibitor with anything that contains tyramine, such as wine and cheese. Choice (D), a drug overdose, is nonspecific, and the question gives no evidence of a bacterial infection, Choice (E).
You’re going to school with a colleague who swears that he has every medical condition you are studying. In school, you’re currently studying the GI system. He’s having some diarrhea and fears he may have colon cancer. He beseeches the physicians to allow him to have a colonoscopy. This behavior could be an example of what psychiatric condition?
(A) Malingering (B) Factitious disorder (C) Somatoform disorder (D) Hypochondriasis (E) Social phobia
(D) Hypochondriasis
You are evaluating a 55-year-old man who comes to the clinic complaining of a fever. You take his temperature, and it is 39.4°C (103°F). You find out that this person may have somehow heated the thermometer with a lighter to induce a false reading. This behavior could be an example of what psychiatric condition?
(A) Malingering (B) Factitious disorder (C) Somatoform disorder (D) Hypochondriasis (E) Social phobia
(B) Factitious disorder
This person is actively trying to mess with the equipment to cause the healthcare provider to think a disease or medical condition is present when it really isn’t. Therefore, he has factitious disorder, Choice (B). This condition differs from Choice (A), malingering, in which the person is making up symptoms or acting sick. With malingering, the person isn’t trying to sabotage medical equipment or do something illegal. Concerning Choice (C), a somatoform disorder is a mental disorder causing physical symptoms; a somatoform disorder is not a syndrome that’s purposely made up. With Choice (D), hypochondriasis, the affected person thinks that he or she has every medical condition. With Choice (E), the person is afraid of a particular aspect of society, such as when someone is afraid to go on stage or be seen in public places.
What’s the most immediate treatment needed in a patient suffering from neuroleptic malignant syndrome?
(A) Intravenous diuretics (B) Intravenous haloperidol (Haldol) (C) Intravenous metoclopramide (Reglan) (D) Warming blanket (E) Dantrolene sodium
(E) Dantrolene sodium
The immediate choice for the treatment of neuroleptic malignant syndrome (NMS) is Choice (E) — you need to do something to stop the muscle rigidity, so you give dantrolene sodium. Bromocriptine can also be given.
Concerning Choice (A), the treatment of choice is intravenous fluids, not diuretics. You don’t want to dehydrate this person. Choices (B) and (C) would be contraindicated: You would not give a neuroleptic to someone with neuroleptic malignant syndrome. Concerning Choice (D), you’d use a cooling blanket, not a warming blanket, because the person would be hyperthermic.
A 46 year-old-man presents to a primary care physician with a chronic cough. He reports that he has not seen a physician in 25 years, but he has not been sick during this time. He lives in a cabin in the woods, and although he does not have any friends, he reports that he does not feel lonely. He is well-groomed, dressed normally, and does not have any bizarre beliefs. He describes his mood as “fine”, and his speech is fluid, albeit limited in amount. His thought process is linear, and his thought content is appropriate. He denies any visual or auditory hallucinations. Which of the following is true about this patient’s psychiatric condition?
A It has a high correlation with schizophrenia
B The patient should be treated with an antidepressant
C No specific treatment is needed at this time
D The patient is likely very responsive to criticism and praise
E The patient likely suffers from a co-morbid substance use disorder
C No specific treatment is needed at this time
= Schizoid personality disorder
This patient’s clinical presentation is consistent with schizoid personality disorder (SPD), which does not require treatment, unless the patient presents with a co-morbid Axis I disorder, such as depression, psychosis, or anxiety.
dressed normally / well groomed / no bizarre beliefs / not lonely when alone / no friends / fluid speech = not schizotypal
No hallucinations = no schizophrenia
Frances, an only child, is a 20-year-old receptionist in a wholesale dress house who feels that she hasn’t been discovered yet by the fashion world. She has done some showroom modeling for customers and is always daydreaming that she will be swept off her feet by the next wealthy man who comes through the door. Eating in the local restaurants, she always takes the opportunity to complain about the clumsy waiters, the inept service, or the poor quality of the food. Frances engages in a rich fantasy life in which she is adored and loved by crowds of onlookers and where she gets even with all the people whom she perceives have been mean to her.
A. Histrionic Personality Disorder B. Narcissistic Personality Disorder C. Borderline Personality Disorder D. Dependent Personality Disorder E. Immature Personality Disorder
B. Narcissistic Personality Disorder
Matt is an active, physically strong, and exceptionally tall 18-year-old 12th grader who stays out late at night with his friends, despite his parents forbidding it. He ran away from home several times when he was younger and is truant from school frequently. His grades are poor and he has been left back and placed in special education classes. The school psychologist gave him a diagnosis of Conduct Disorder when he was 1C. When he attends school now, he threatens and intimidates students and teachers and seems to enjoy getting into fights. He has attacked one student with a brick and boasts that he carries a knife and has a gun hidden on school grounds. Matt has never expressed feelings of remorse for his actions. A good talker, he often engages in conning his teachers and enjoys the attention it seems to get from the other kids. A favorite weekend activity in which Matt has been engaging for the past several years is shoplifting. Your diagnosis would be:
A. Oppositional Defiant Disorder
B. Conduct Disorder
C. Antisocial Personality Disorder
D. Disruptive Behavior Disorder Not Otherwise Specified (NOS)
E. Attention-Deficit/Hyperactivity Disorder (NOS)
C. Antisocial Personality Disorder
A 32-year-old Hispanic man comes to your general practice office for the first time. In the course of taking his history, the man mentions several times that he feels exploited by his employees and that he doubts their loyalty to his company. “In fact,” he says, “I have even overheard them talking about how they would change the company if I were no longer in charge.” He says he is consequently reluctant to confide in others, especially in his business partner, and he even wonders if he should be telling you all this since you have just met. Based on this information alone, you might suspect that his man is suffering from which personality disorder?
A. Paranoid Personality Disorder B. Schizoid Personality Disorder C. Schizotypal Personality Disorder D. Antisocial Personality Disorder E. Histrionic Personality Disorder
A. Paranoid Personality Disorder
A 30-year-old woman is brought to the emergency department following a minor car accident. Physical examination is unremarkable except for a small laceration above her eyebrow. As the physician is talking to the woman she becomes very emotional and theatrical when describing the car accident, but would not provide further details about the accident other than talking about how it was a “near death experience.” As the physician proceeds with the examination, the patient becomes less preoccupied with her “near death experience” and very provocative toward the examiner. Her toxicology screen is negative. Her past psychiatric history is negative for depression and any suicide attempts. She is single and lives alone but states that she has been in several intimate relationships. She works in theater and has been in several local plays. Which of the following is the most likely diagnosis?
A Borderline personality disorder B Dependent personality disorder C Histrionic personality disorder D Narcissistic personality disorder E Schizotypal personality disorder
C Histrionic personality disorder
Histrionic personality disorder is characterized by a pervasive pattern of excessive emotionality and attention seeking and includes features such as inappropriate sexual behaviors, shallow or exaggerated expression of emotion, and consistent use of physical appearance to draw attention to self.
A 32-year-old man comes to the emergency department because of multiple non-lethal stab wounds. He is currently imprisoned for murder and armed robbery. After his condition stabilizes, he insists on staying in the hospital until he is fully cured. He threatens to harm the attending physician if she endorses his return to prison. Which of the following is his most likely diagnosis?
A Adjustment disorder with mixed features B Antisocial personality disorder C Bipolar disorder, manic D Borderline personality disorder E Narcissistic personality disorder
B Antisocial personality disorder
Antisocial personality disorder is characterized by a pervasive pattern of disregard and violation of the rights of others and the rules of society
This patient’s history of criminal activity, evidence of recent physical altercation, and acting out with threat of harm toward the attending physician is most suggestive of antisocial personality disorder. Patients with antisocial personality disorder demonstrate a pervasive lack of empathy and remorseless disregard toward the rights and well-being of others in their behavior. They often become involved in criminal activities and act in an impulsive and irresponsible fashion.
A 25-year-old woman is brought to the emergency department after telling her roommate that she wants to just “go away” following an argument with her boyfriend. Her roommate states that the patient has few close friends and often believes that her relationships are more intimate than they actually are. She is also easily influenced by her friends and often is perceived as being shallow. She denies depressed mood, grandiosity, weight changes, sleep disturbances or suicidal ideation prior to the argument. What other trait is most likely present in this patient?
A A grandiose sense of self-importance
B A style of speech that is excessively impressionistic and lacking in detail
C Needs others to assume responsibility for most areas of her life
D Transient, stress related paranoia
E Views herself as socially inept
B A style of speech that is excessively impressionistic and lacking in detail
= HISTRIONIC PERSONALITY DISORDER
CLINICAL MANIFESTATIONS
⦁ Self-absorbed
⦁ “temper tantrums”
⦁ efforts to draw attention to themselves with the need to be the center of attention
⦁ often inappropriate, sexually provocative, seductive with shallow or exaggerated emotions
⦁ seek reassurance + praise often
⦁ may believe their relationships are more intimate than they actually are
⦁ can be suggestible = easily influenced by others or circumstances
A 28-year-old man comes to the emergency department because of a recent suicide attempt. He was found after taking an unknown amount of sleeping pills. He recently lost his job, and has feelings of low self-worth. Six months ago, he was seen by a psychiatrist for episodes of self-mutilation due to feelings of being “empty” and poor self-esteem. Over the last three years, he has accumulated debt from excessive gambling, and has been forced to sell his home and car. What defense mechanism is this patient most likely to display during his hospital stay?
A Dissociation B Sublimation C Splitting D Acting out E Displacement
C Splitting = Borderline Personality Disorder
Patients with borderline personality disorder commonly use the defense mechanism of splitting. Splitting is defined as labeling people as all good or all bad. For example, he may only express his concerns to his nurses, but may view all doctors as mean and non-understanding of his needs.
Borderline personality disorder is a pattern of abnormal behavior with extreme fear of abandonment, and unstable relationships with other people. This pattern of behavior typically begins in early adulthood. Substance abuse, depression, and eating disorders often co-exist with borderline personality disorder
A 20-year-old woman comes to the clinic because of feelings of emptiness and the desire to self-harm. Physical examination and laboratory tests show no abnormalities. A clinical assessment is performed and she is diagnosed with borderline personality disorder. Which of the following groups of characteristics would most likely be seen in this patient?
A Unstable self-image, impulsivity, suicidal behavior
B Arrogance, feelings of grandiosity, lack of empathy
C Lack of personal relationships, promiscuity, splitting behavior
D Attention seeking, inappropriate sexual seduction, rapidly shifting emotions
E Intense interpersonal relationships, rapidly shifting emotions, submissive
A Unstable self-image, impulsivity, suicidal behavior
Patients with borderline personality disorder are unstable and have intense interpersonal relationships, frantic efforts to avoid abandonment, unstable self-image, impulsivity that leads to self-damaging behavior, such as sexual promiscuity or substance abuse, suicidal behavior, rapidly shifting affect, intense anger, or psychotic features.
An 18-year-old woman comes to the emergency department because her mother states that “she just can’t deal with her anymore.” The patient is reserved and reluctant to answer questions. Her mother says her daughter regularly abuses alcohol, and has many sexual partners. The patient says her most recent boyfriend is “the love of her life and would do anything” for her. She does not have a job and is home schooled, due to an inappropriate relationship with a former teacher. Physical examination shows numerous cuts on her forearm. Which of the following is most likely present in this patient?
A Obsessive compulsive personality disorder B Histrionic personality disorder C Schizoid personality disorder D Borderline personality disorder E Narcissistic personality disorder
D Borderline personality disorder
Borderline = hallmarked by a pattern of instability in relationships, self-image, identity, behavior, and affects often leading to self-harm and impulsivity. Substance abuse is common. Self cutting is also very common.
A 25-year-old woman comes to the emergency department because of cuts she made with a razor blade to her left forearm and right hip. She says her boyfriend broke up with her recently, and she has been feeling very lonely and empty. She reports frequent cutting with a razor blade on herself when feeling this way in the past. She lives with a roommate, but reports constant fighting with her. Which of the following is the most likely diagnosis?
A Bipolar disorder type II B Borderline personality disorder C Dysthymic disorder D Major depressive disorder E Schizoid personality disorder
B Borderline personality disorder
Borderline personality disorder is characterized by instability of interpersonal relationships, self-image, and emotions, as well as by impulsivity across a wide range of situations, causing significant impairment or subjective distress.
A 23 year-old man comes to the psychiatry clinic, accompanied by his mother, because she is concerned after seeing cut marks on his arms. His mother says that his life is chaotic and unstable. He often brings nice girls home whom he “likes a lot” to meet his family, only to come home days later, slam his bedroom door shut, and yell that he “hates all of them” and “just wants to be alone”. He has difficulty keeping jobs because of his angry outbursts, but also has no trouble finding a new one because he can be very friendly and professional in interviews. Which of the following is the most likely diagnosis?
A Schizotypal personality disorder B Antisocial personality disorder C Avoidant personality disorder D Histrionic personality disorder E Borderline personality disorder
E Borderline personality disorder
Borderline personality disorder is characterized by instability of interpersonal relationships, self-image, and emotions, as well as by impulsivity across a wide range of situations, causing significant impairment or subjective distress.
While this patient’s ability to charm in interviews could point to antisocial personality disorder, and his emotional outbursts could point to histrionic personality disorder, the constellation of symptoms (self-mutilation, mood instability, and splitting with regard to girls) point to borderline personality disorder.
A 23-year-old woman is urgently reviewed by the psychiatric crisis team because of a suicide attempt an hour ago. Her mother states that she found her daughter on the floor cutting her wrists because her boyfriend broke up with her today. Medical history includes childhood sexual abuse and multiple previous admissions for suicide attempts. The patient states that she is in a good mood today because the man delivering her hospital food gave her an extra jello cup and she believes that he is in love with her. She states that he is “so much better than her ex-boyfriend”. Examination shows the patient sitting up in bed applying makeup. There are multiple scars on her wrists from previous suicide attempts. Which of the following personality disorders is this patient most likely exhibiting?
A Dependent B Narcissistic C Histrionic D Borderline E Avoidant
D Borderline
Borderline personality disorder is a DSM-V diagnosis characterized by seeking approval from others for self-identity, impairments in empathy and intimacy, and pathological personality traits such as grandiosity and attention seeking.
Patients with borderline personality disorder have instability in their interpersonal relationships, self-image, and mood. They also often exhibit impulsive behavior. The gender ratio for this disease is 3:1 with more females being affected. The onset of the disease typically is in adolescence and early adulthood. There is usually a history of childhood trauma associated with borderline personality disorder. Patients typically have comorbid diseases such as depression, anxiety, substance abuse, and eating disorders. They typically have a history of suicide attempts or threats which are meant to bring on attention or get what they desire. On physical exam there is often multiple burns or markings from past suicide attempt. They also tend to see people as all “good” or all “bad,” and this is a psychiatric phenomenon known as splitting.
A 32-year-old female is caught by the police because of identity theft. When confronted by the police, she lies about the accusations and tries to run. She also tries to escape custody by putting the lives of the police officers in danger. Which of the following personality disorder is she displaying?
A Schizotypal personality disorder B Borderline personality disorder C Schizoaffective personality disorder D Schizoid personality disorder E Antisocial personality disorder
E Antisocial personality disorder
Antisocial personality disorder is always characterized by a history of conduct disorder before the age of 15. If left untreated, this will cause antisocial personality disorder.
The DSM-V defines antisocial personality disorder as failure to conform to social norms with respect to lawful behaviors, as indicated by repeatedly performing acts that are grounds for arrest; deceitfulness, as indicated by repeated lying, use of aliases, or conning others for personal profit or pleasure; impulsivity or failure to plan ahead; irritability and aggressiveness, as indicated by repeated physical fights or assaults; reckless disregard for the safety of self or others; consistent irresponsibility, as indicated by repeated failure to sustain consistent work behavior or honor financial obligations; lack of remorse, as indicated by being indifferent to or rationalizing having hurt, mistreated, or stolen from another; the individual is at least age 18 years; there is evidence of conduct disorder with onset before age 15 years; and the occurrence of antisocial behavior is not exclusively during the course of schizophrenia or bipolar disorder.
A 17-year-old girl comes to the emergency department with her parents because she inflicted multiple cuts on her forearms once her boyfriend left her. Her primary physician is notified, he arrives to her room and she says: “I am so happy that you are here, since you are the only person in this world that gets me.” The next day, the doctor goes to follow-up the patient’s progress. As he tries to talk to her, the patient says: “I will not confide in you anymore because you do not care at all about what is happening to me.” She also adds that, “The nurses are fantastic and they take really good care of me.” What type of defense or coping mechanism is this patient most likely using?
A Denial B Displacement C Projection D Regression E Splitting
E Splitting
Splitting is a primitive coping mechanism and the key symptom for patients of borderline personality disorder, such as the one described in this clinical vignette. Splitting is an ego mechanism that fluctuates between idealization and devaluation of objects, actions, and persons.
Splitting is an ego mechanism that fluctuates between idealization and devaluation of objects, actions, and persons. Individuals who use splitting as a coping mechanism tend to think in extremes. In psychoanalytic theory, people with borderline personality disorder are not able to integrate the good and bad images of both self and others. This primitive coping mechanism is the key symptom for patients of borderline personality disorder such as the one described in this clinical vignette. Splitting follows “All or None” phenomenon. The doctor is very good one day, and becomes very bad another day. It’s most commonly seen in borderline personality disorder.
A 27-year-old woman and her boyfriend of 2 years are referred to couples counseling. The boyfriend is thinking of proposing, however there has been strain on their relationship because of the patient’s inappropriate behavior. The boyfriend states that she is constantly seeking his attention, and “throws a fit” in public if she feels he is not. In addition, she dresses like a “high school cheerleader” and flirts with all of his friends and even his own brother. Most recently she accompanied him to a work dinner and started talking openly about their sex life, which caused him a great deal of embarrassment. When asked about these allegations, the patient started crying profusely and stated that her boyfriend hates her and is ashamed of her. In effort to calm her down, the therapist patted her arm, after which she began flirting with him. Which of the following is the most likely diagnosis?
A Borderline personality disorder B Dependent personality disorder C Histrionic personality disorder D Schizoid personality disorder E Schizotypal personality disorder
C Histrionic personality disorder
Histrionic personality disorder is characterized by a pervasive pattern of excessive emotionality and attention seeking and includes features such as inappropriate sexual behaviors, shallow or exaggerated expression of emotion, and consistent use of physical appearance to draw attention to self.
A 25-year-old woman comes to the psychiatrist because of a referral by her family physician. She has been treated for a myriad of oral and genital sexually transmitted infections over the past five years, and she says she has had several hundred sexual partners in her lifetime. When the psychiatrist enters the room, she immediately jumps out of her chair and gives him a hug, telling him she is excited to see him. She is dressed provocatively, and during the interview, she strokes the psychiatrist’s shoulder and winks at him. She tells him that she frequently goes out alone to the local bars, dances on the tables, and often performs sexual favors for male customers in the restrooms, suggesting that she would do the same for him. Which of the following is the most likely diagnosis based on the patient’s presentation and medical history?
A Antisocial personality disorder B Avoidant personality disorder C Borderline personality disorder D Histrionic personality disorder E Narcissistic personality disorder
D Histrionic personality disorder
Histrionic personality disorder is characterized by attention-seeking behavior, excessive emotionality and excitability, attention seeking, sexual provocation, and excessive concern with appearance.
A 12-year-old comes to the hospital because of a repetitive pattern of aggressive behavior. He has had episodes of peer-fights, theft, and vandalization without remorse for the past year. He was caught torturing his neighbor’s cat. Compared to his peers, which of the following is he most likely to develop as he gets older?
A Oppositional defiant disorder B Delusional disorder C Conduct disorder D Antisocial personality disorder E Anxiety disorder
D Antisocial personality disorder
AS HE GETS OLDER!
patient has conduct disorder right now (12 = can’t have antisocial yet) - but can develop antisocial in the future
A 30-year-old woman comes to the emergency department because of an attempt of suicide. She took an entire bottle of acetaminophen after breaking up with her boyfriend. She is tearful and shouts that she has nothing to live for since she lost the love of her life. She was in the relationship for three weeks. She is convinced that he left her for her best friend. One year ago, she was seen by a psychiatrist for episodes of self-mutilation due to feelings of being lonely and having no self-worth. Over the last five years, she has been accumulating debt, and had recently needed to sell her home and move in with her sister. Despite this debt, she continues to buy new clothes, and goes out to the bar most nights of the week. Which of the following is the most likely explanation for her behavior?
A Dependent personality disorder B Narcissistic personality disorder C Antisocial personality disorder D Histrionic personality disorder E Borderline personality disorder
E Borderline personality disorder
Patients with borderline personality disorder have unstable moods and relationships. These patients tend to be impulsive and it is common for them to have a history of multiple suicide threats or attempts.
A 25-year-old medical student comes to the psychiatric clinic because of difficulties working in groups over the past year. While in the waiting room, he demands to be seen by the chair of psychiatry himself because he has “a reputation to uphold.” During the visit, he explains that he refuses to cooperate on any group projects with other students because he insists that he can do the work better himself. He further notes that he wants to specialize in experimental neurosurgery and can’t have other students dragging down his grades. He reports that he often enters professors offices or the faculty lounge uninvited to share personal anecdotes and “fascinating” stories from his research. He also states that he has no issues with exploiting his classmates for his own gain because medical school is “inherently competitive.” Which of the following disorders does this patient most likely have?
A Narcissistic personality disorder B Borderline personality disorder C Paranoid personality disorder D Histrionic personality disorder E Antisocial personality disorder
A Narcissistic personality disorder
Narcissistic personality disorder is a cluster B disorder. These patients have an inflated sense of self, sense of entitlement, and lack of regard for others.